You are on page 1of 78

Can exercise really improve your mental health?

Yes, studies have shown that exercise brings


about both short- and long-term psychological enhancement and mental well-being. Some of the
psychological benefits from physical activity include improvement in self-confidence, relief of
tension and feelings of depression, positive changes in mood, increased alertness, clearer
thinking, and positive coping strategies. Individuals of all ages and gender can realize these
benefits from exercise.

Are those so-called "sports drinks" beneficial during exercise? The composition of these
drinks is basically water, electrolytes (minerals capable of carrying an electrical charge), and
glucose. Sweat consists mostly of water and electrolytes. In prolonged endurance events, glucose
(carbohydrate) replacement may be beneficial. Also, endurance exercise in heat contributes to
heavy losses of water and electrolytes which need to be replenished.

Is it O.K. to drink water while exercising? Yes, your body's circulation system must get food
and nutrients to the working cells to carry out their chemical reactions. Sweating during exercise
depletes your body's water supply, which may lead to dehydration. Do not depend on your thirst
to tell you to drink water. Try to drink at least 8 ounces of cool water for every 30 minutes of
vigorous exercise. Use that water bottle!

What are free radicals and antioxidants? Free radicals are unstable molecules produced by
chemical reactions utilizing oxygen in the body's cells. A variety of external factors can promote
free radical formation including smoking, drinking alcohol, and pollution. Antioxidants (vitamin
C, E, and beta carotene—a precursor to vitamin A) protect the cells from free radicals by
neutralizing the process of molecular oxidation that leads to their formation.

Does the distribution of body fat have any health consequences? Yes, people who gain fat in
the abdominal area have a higher risk of coronary heart disease, high blood pressure, diabetes
and stroke as compared to individuals who gain fat in the hip area. Men tend to gain fat around
the waist and women at the hip.

What are the leading causes of death in the United States? According to the National Center
for Health Statistics, they are heart disease, cancers, strokes, injuries, chronic lung diseases,
pneumonia, diabetes, suicide, AIDS, and homicide.

Is it O.K. to drink beer after working out? Alcohol is a diuretic, which means that it
stimulates urine production. Following a workout you want to replenish your body with lost
fluids. Consequently, drinking any alcohol beverage before or after exercise is not
recommended.

Do you burn more calories when you are hot or cold, or does it even make a difference?
Shivering in clold weather requires more energy but not enought to enhance a weight loss
program.

What are the limiting factors of flexibility? With the muscles relaxed, and reflex mechanisms
minimally involved, researchers have found the relative contributions of soft tissue to joint
stiffness to be the following: joint capsule, including ligaments (47%), muscles and their fascial
sheaths (41%), tendons (10%), and the skin (2%).
Why does the mirror show your change of shape before the scale? Muscle is denser than fat.
A pound of fat bulges out 18% more than a pound of muscle. Since you are adding muscle to
your body as you shape up, you will often notice a loss of inches before a loss of weight.

Why do people who exercise have fewer colds and viral infections? It has been observed that
exercise may boost the immune system and that physically active people take better care of
themselves. It should be noted that regular, strenuous exercise has also been shown to have an
opposite effect on the body's immune system. This adds support for an exercise prescription
directed towards regular, moderate levels of exercise participation.

Why do you perspire more after you stop working out? During exercise your muscles need
most of the blood to get oxygen for the activity. Upon the cessation of exercise, more blood is
diverted to the skin to cool the body by means of sweat production. Also, during most modes of
exercise you are moving allot, which helps sweat evaporate more efficiently during the activity.

What is cholesterol? Cholesterol is a fat-like substance used to help build cell membranes,
make some hormones, synthesize vitamin D, and form bile secretions that aid in digestion. Since
fat can't mix with water, which is the main ingredient of blood, cholesterol's most important job
is to help carry fat through your blood vessels. Before cholesterol can enter the bloodstream it is
coated with a protein, referred to as a lipoprotein. Lipoproteins are transport vehicles in the
circulation plasma that are composed of various lipids such as cholesterol, phospholipids,
triglycerides and proteins known as apoproteins. The major classes of lipoproteins are
chylomicrons, very low-density lipoprotein cholesterol, low-density lipoprotein cholesterol and
high-density lipoprotein cholesterol.

Which is the 'bad' and 'good' cholesterol? The low-density lipoprotein cholesterol (LDL-C) is
the primary transport carrier of cholesterol in the circulation. It is referred to as the 'bad'
cholesterol because too much cholesterol, from eating foods high in saturated fat, often leads to
LDL-C pieces adhering to the inner walls of the blood vessels, narrowing the blood passages. On
the other hand, the high-density lipoprotein cholesterol's (HDL-C) primary function is to
transport cholesterol from the tissues and blood to the liver for excretion or recycling. It is
referred to as the good cholesterol.

Are natural vitamins better for you than manufactured vitamins? No, your body can't
distinguish the difference between vitamins manufactured in a laboratory and natural vitamins
extracted from food.

Does cold weather give you colds? People exposed to chilling temperatures often feel they are
more susceptible to colds. However, it is viruses that give you colds, not the weather. Colds are
actually more common in cold weather because people are inside more, and thus more exposed
to germs.

What's a good substitute for those high-fat potato chip snacks? Try pretzels. They are almost
fat-free. As a matter of fact, pretzels are one of the fastest growing snack foods in the U.S.
However, there is good news for you 'chip' lovers. There are now some new and improved chips
that are low in sodium, fat and calories. Read the labels at your supermarket. Look for the chips
that are baked, not fried, and avoid chips made in hydrogenated oil. Remember to check the total
fat content and serving size, too.
Can you drink too much caffeine? Most people have internal regulators that tell them when to
stop drinking caffeine. However, drinking 4-6 cups of coffee a day may result in symptoms
referred to as caffeinism: breathlessness, headache, lightheadedness and irregular heartbeat. Too
much caffeine may also trigger a panic attack. For college students, caffeine-containing soft
drinks have overtaken coffee as the primary source of caffeine. Most experts advice no more
than 200 milligrams of caffeine consumption, which is about the same as two to three 5-ounce
cups of coffee.

Are these low-carbohydrate, high-protein diets any good? Initially these diets help people
lose weight because the body loses water; approximately 3 parts of water to 1 part of
carbohydrate. A low carbohydrate diet is fairly unappetizing. However, the body cannot handle
the extra amounts of proteins and will end up storing much of it as fat.

How do you avoid repetitive strain injuries? With the wonderful benefits of computer use
there are also some physical perils to know. Repetitive strain injury (RSI) results from hand,
wrist, arm and neck injuries from a repetitive (fast) work. Carpal tunnel syndrome is a form of
RSI which results from pain in the tendons and nerves across the wrist. To avoid, take frequent
breaks during your typing and make sure your computer set-up avoids any neck, back, wrist and
eye strain (see Sitting at your Computer Station).

Are vegetarian diets more healthful than diets that include animal foods? Vegetarian diets
are higher in fiber and lower in saturated fats and cholesterol. Studies also show that vegetarians
suffer less from high blood pressure, diabetes, heart disease and obesity. However, many
‘vegetarian foods’ such as chips and candy contain allot of fat and calories, with little nutrient
value. Also, vegetarians need to be conscious of deficiencies in iron, calcium, vitamin D, vitamin
B12, and zinc.

What are triglycerides? Triglycerides (TG) are fats that circulate in the bloodstream that
provide energy for the body. It is uncertain whether high TG levels are associated with coronary
heart disease. However, high TG levels are associated with diabetes, kidney diseases, and
obesity. Steps to lower TG levels include cutting down on saturated fat, losing weight, exercise,
and quitting smoking.

What is this creatine supplement craze? Creatine is an amino acid found predominantly in the
muscles in the form of creatine phosphate, where if facilitates energy production. It also helps to
reduce the lactic acid accumulation during intense exercise. Most investigations have shown that
creatine supplements boost short-term muscle strength and power. However, it is important to
note that the long-term effects of high doses of creatine are unknown.

Are the sunless tanning lotions safe? According to the U.S. Food and Drug Administration
they are safe. The main ingredient in self-tanning products is dihydroxyacetone, which reacts
with the top layer of skin to form a light brown tan stain. Remember that these lotions do not
give you any sun protection, however.

How successful are all the stop smoking interventions? The interventions range from
acupuncture, nicotine gum and patches, hypnotism, anti-depressants and behavior modification
approaches. None of the methods has outstanding long-term effects, with each method showing a
20% success rate after one year. Quitting smoking is essential. A person needs the support of the
family and friends as well as a sustained effort to find the intervention that works.

What causes your limbs to "fall asleep?" A limb will become numb when you remain seated
or lying in a position that compresses a nerve in the limb or stops the blood flow to the nerve.
This sometimes happens when you cross your legs or arms for an extended period of time. When
you remove the pressure the tingling is the nerve sensitivity returning. Moving the limb helps to
speed up the recovery. Men who carry a thick wallet in their back pocket (with tight pants) can
put pressure on the sciatic nerve, causing a similar numbness and pain.

Will exercise reduce a woman’s risk to breast cancer? Exercise reduces the risk of heart
disease, diabetes, stroke and obesity. It appears to have a protective effect against breast cancer
but more research is needed. However, the best advice for men and women is that they should
exercise.

What dietary substances are needed to prevent osteoporosis besides calcium and vitamin
D? Two other substances of importance are magnesium and potassium which are both found in
fruits, vegetables, milk and whole grains.

Is it true that when you eat out, you usually eat more calories than at home? Yes, large
portion sizes and high-fat entrees burden most of this responsibility. Be aware that when you go
out you are less likely to eat nutritious food. Forego eating appetizers as they add to the calories.
Perhaps split a desert as opposed to ordering one for yourself. Also, most Americans neglect
fruits and vegetables when eating out.

I read a recent study that said high-fiber diets don’t cut colon cancer? Even if the study was
well-conducted research, it is still only one study. Decades of research suggest that high-fiber
foods are a protection against colon cancer. Remember, stay away from those sugary sweets that
are consistently associated with colorectal cancer.

I read allot about the health benefits of soy. Are they true? Soy has been shown to be very
health beneficial. For instance, research shows that soy consumption will lower triglycerides and
LDL-cholesterol (lousy cholesterol) in people with high blood lipids, while raising the HDL-
cholesterol (helpful cholesterol). Note also that heart disease is much lower in Asian countries,
where soy is a dietary staple. Soy also has been shown to play a role in lowering breast and
uterus cancer.

How does soy improve one’s health? It is felt that the hormone phytoestrogen mimics some of
the effects of estrogen, actually blocking off some of the harmful effects associated with
estrogen. Also, remember soy foods are low in saturated fat and are cholesterol free. For an
internet source on soy-based foods and products go to www.soyfoods.com/

Is it true that tea may possibly be a healthful beverage? Yes, black and green tea (sorry, not
herbal teas) appears to lessen cholesterol’s damaging affect on your arteries, as well as protect
against cancers of the skin and gastrointestinal tract. Make sure you step tea for 3 minutes for the
beneficial antioxidants in the leaves to enter in the beverage.

How effective is the supplement creatine? Creatine is made by your body and supplied in
foods such as fish and vegetables. It is beneficial when your muscles need short, quick spurts of
energy for activities like resistance exercise and sprinting. However, the weight gain experienced
by some individuals may actually be more attributable to an uptake of water by the muscle cells.

What are the effects of removing the cool-down from your exercise routine? The purpose of
the cool-down is to restore all cells,tissues and organs of the body that have been stressed (or
challenged) from the exercise workout. Removing the cool-down removes this process and thus
hinders the body’s ability to restore itselt to homeostasis.
I am so rushed to get things done I am contemplating taking some of these liquid meal
replacements on a regular basis. Many of these liquid meals provide plenty of calories as well
as a number of minerals and vitamins. But typically they do not provide the health-promoting
fiber and phytochemicals that are found in fruits and vegetables. So, this is not to discourage
their consumption, but make sure you balance the intake with real meals.

What does it mean when a food has the American Heart Association logo on it? The red
heart with white check mark means no more than 3 grams of fat, 20 milligrams of cholesterol,
and 480 milligrams of sodium. The food must also have at least 10% of the daily value for one or
more of these nutrients: protein, vitamin A, vitamin C, calcium, iron, or dietary fiber.

As you age, is there anything you can do to slow the decline in mental PROCESSES? The
degree of decline varies from person to person, but the age-related changes are due to alterations
in the brain’s frontal lobe, right behind the forehead. However, new research suggests that
sedentary older people who take up aerobic exercise (such as walking) can slow the loss in
mental agility even if they never exercised before in their lives. Researchers believe that the
aerobic activity improves mental functioning by increasing the supply of oxygen to the brain.

How can I, as a young man, protect against prostate cancer? Very wise preventative
measure. Prostate cancer is the second leading cause of cancer in American men. Research
confirms that diets high in fat, calories, and animal products are strongly associated with this
deadly disease. Diets high in grains, cereals, soybeans, nuts and fish, on the other hand, appear to
have protective effects.

What’s the final scoop: Butter or Margarine? Good question. While margarine is indeed
lower in saturated fat than butter, it’s higher in trans fatty acids, which do contribute to high
levels of cholesterol. Recent research does suggest that margarine is healthier. Also, it appears
that soft tub margarine may be the best overall choice.

What are the most common types of arthritis? There are actually more than 100 different
forms. Osteoarthritis is a wearing and tearing form that affects the fingers and weight-bearing
joints (knees, back, hips). Rheumatoid arthritis causes irritating joint stiffness and swelling.
Fibromyalgia leads to pain at different points of the body as well as insomnia, morning stiffness
and constant fatigue. Gout is caused when uric acid accumulates in the joints, specifically the big
toe, knees and wrists. Lupus can damage the kidneys, heart, skin, lungs and joints.

What is the most common nutritional deficiency in the U.S? Iron deficiency. It affects some 8
million women of childbearing age and upwards of 700,000 toddlers.

I only have time right before bed to exercise and am worried that it will impair my evening
sleep patterns. Interestingly enough, recent research has shown that vigorous exercise ending 30
minutes before bedtime failed to disrupt sleep.

I have trouble sleeping. Do you have any suggestions to get a better night’s rest? Try to
establish a sleeping schedule of going to bed every night and getting up at the same time each
morning. Work on the stress levels in your life as this can dramatically affect sleep. You sleep
better if the room temperature is slightly cooler than other parts of the house. Try to avoid
drinking too many fluids before bedtime so that your bladder isn’t full. Remember, spicy foods
for dinner, or as snacks, may cause heartburn or stomach acidity and impair your quality of
sleep. Also, avoid alcohol near bedtime. While it may help you fall asleep, when the alcohol
wears off your brain actually becomes more alert. Finally, stay away from sleeping pills as they
may lead to unwanted side effects and health-related risks.

I use a computer everyday. Are there any exercises that will help prevent and control
carpal tunnel syndrome? Carpal tunnel syndrome is a condition that develops from repetitive
wrist motions such as typing at a computer terminal. This motion may lead to pressure on the
median nerve in the wrist, and great pain and discomfort. Do the following exercises throughout
the day. Frequently take your wrists through a complete range of motion in both directions.
Make a tight fist with both hands, hold for 6 seconds, and repeat 3 to 4 times. Also, circle your
wrists in both directions while holding your hands in a fist. Make sure you take short breaks from
your typing to do these exercises (and to rest your wrists) in order to safeguard from developing
carpal tunnel syndrome.

What causes muscle cramps? New research on exercise-associated muscle cramping suggests
the cramp occurs as a result of abnormal nerve activity from the spine, probably related to
fatigue. Although not well understood, it is believed that tired muscles going through repeated
shortening contractions are more vulnerable to cramping. Avoiding over fatiguing workouts and
incorporate regular stretching to best ward off muscle cramps.

What is this new concept of training called periodization? Although not really new, it is now
being used regularly with recreational resistance training enthusiasts. Periodization is most
widely used in resistance training and involves systematically alternating high loads of training
with decreased loading phases in order to improve components of muscular fitness. The system
is typically divided into three cycles: 1. The microcycle, which lasts up to seven days 2. The
mesocycle, which can be from two weeks to a few months, is subdivided into preparation,
competition, peaking and transition phase. 3. The macrocycle is the overall yearly training
period.

What is ‘carbo loading’ and how do you do it? Carbo loading is a method of super saturating
your muscles with glycogen. It has been shown to improve endurance performance in events
lasting over 90 minutes and is often used in competitive events. About seven days out from the
event, begin by eating a diet high (60% to 70%) in high-glycemic carbohydrates such as rice,
pasta and potatoes. Make sure you are getting adequate amounts of fat and protein as well. Drink
plenty of water and increase your water intake four to eight cups (ABOVE NORMAL) about two
days before the event. Avoid dehydrating drinks and foods. As the event comes closer, remember
to taper down your training so you body will be appropriately rested for the contest.

What does the term ‘MET’ mean? This term is used to describe energy expenditure of an
activity. One MET is equivalent to the energy expenditure of a person at rest. It is expressed in
terms of oxygen uptake (i.e., 3.5 ml O2/kg/min). It is very much like a shorthand method of
describing energy requirements. For instance, running 6 miles per hour is about 10 METs while
walking 3 miles per hour is about 3.3 METs.

What are phytochemicals? They are not vitamins. They are not minerals. Phytochemicals, are
plant chemicals that offer great health benefits. The have been shown to project against heart
disease, cancer, diabetes, osteoporosis, and other medical conditions. The only way to get
phytochemicals is to eat or drink them in fruits, vegetables, juices, nuts and whole grain
products.

Is Yo-Yo dieting risky for you? Let’s face it, many well-intentioned diets do fail, which has
concerned researchers that the losing and regaining of weight might be harmful to the body.
However, new research with high blood pressure, which is a major risk factor for heart disease,
shows that Yo-Yo dieting doesn’t appear to have any physiological damage. However, in terms
of emotional wellness, Yo-Yo dieting may be most detrimental.

What is hypoglycemia? Hypoglycemia means low blood sugar. Most individuals may be
affected by what is referred to as reactive hypoglycemia, a condition resulting from glucose
levels dropping after a meal or when you haven’t eaten for several hours. Symptoms include
dizziness, anxiety, shaking, and uneasiness. Eating small, but frequent meals and avoiding
concentrated sources of sugar is recommended.

Can you tell me in minutes or days how much smoking reduces life? According to some
research, every cigarette a man smokes reduces his life by 11 minutes. Each carton of cigarettes
represents a day and a half of lost life. For every year a man smokes a pack a day, he shortens his
life by about two months.

Why don’t diets work? Most diets, especially fad diets, are poorly designed plans that
unrealistically restrict caloric intake. People lose weight initially, but these plans are not
permanent weight loss eating strategies which people can incorporate into a regular lifestyle.
Consequently, INDIVIDUALS usually return to their previous eating habits that encouraged the
weight gain in the first place. Eating habits (and exercise) must be changed permanently for diets
to be successful.
To some degree, the answer depends on what type of workout you are about to do. The typical
30 minute workout of easy jogging or brisk walking really doesn't have specific macronutrient
requirements. However, if your workout is going to be more intense or enduring, here are Some
suggestions. To keep “fueling your metabolic engine” you need more complex-carbohydrates a
few hours prior to the workout. Your goal is to help maintain your body's blood glucose and
carbohydrate (glycogen) stores in the muscles. Also, make sure you drink fluids before the
workout. ACSM recommends 14 to 20 ounces of fluid two to three hours before the event and
possibly another 7 to 10 ounces right before an endurance event.

How can older people improve their memory?


The University of California, Berkely Wellness Letter cites recent research that shows that
aerobic exercise can help sharpen the memory and reduce depression in men and women aged 50
to 77. The aerobic exercise helps to pump the blood more efficiently though the body and brain.
Results show that meaningful results can occur in as little as 4 months.
Should I eat before I workout?
To some degree, the answer depends on what type of workout you are about to do. The typical
30 minute workout of easy jogging or brisk walking really doesn't have specific macronutrient
requirements. However, if your workout is going to be more intense or enduring, here are Some
suggestions. To keep “fueling your metabolic engine” you need more complex-carbohydrates a
few hours prior to the workout. Your goal is to help maintain your body's blood glucose and
carbohydrate (glycogen) stores in the muscles. Also, make sure you drink fluids before the
workout. ACSM recommends 14 to 20 ounces of fluid two to three hours before the event and
possibly another 7 to 10 ounces right before an endurance event.

LECTURE 3
Copyright © 2000 by Bowman O. Davis, Jr. The approach and organization of this material was
developed by Bowman O. Davis, Jr. for specific use in online instruction. All rights reserved. No part of the
material protected by this copyright notice may be reproduced or utilized in any form or by any means,
electronic or mechanical, including photocopying, recording, or by any information storage and retrieval
system, without the written permission of the copyright owner.

FLUIDS AND ELECTROLYTES


Review of Lecture 2
By this time, you should be comfortable with the three fluid compartments of the
body and the partitions that separate them and control their compositions. The roles of
hydrostatic and osmotic pressures in the shifting of fluids from one compartment to
another should be “second nature” to you. You should have also practiced working
through the basic physiology underlying a number of fluid imbalances and you should be
capable of assessing a client experiencing a fluid imbalance.
Now it is important to realize that fluids of the body do not exist independently.
They have a number of chemical or cellular elements dissolved or suspended within
them. Electrolytes, capable of conducting electrical current, exist dissolved in the fluid
compartments of the body. It is because of these dissolved electrolytes and their capacity
to conduct current, that it is possible to measure brain activity (EEG) as well as heart
(EKG) and other muscle (EMG) by placing electrodes on the body surface. However,
electrolytes also serve major physiological functions which are in danger of failing
should an electrolyte imbalance occur.
Continue your text reading through Chapter 10 before proceeding.
Normal Electrolyte Functions
By chemical definition, electrolytes are charged particles (ions). These particles
exist dissolved in the various fluid compartments of the body (intravascular, interstitial,
and intracellular) and perform a variety of functions in the total physiology of the human
body. The electrolytes of importance at this point in the course are: (1) Sodium; (2)
Potassium; (3) Calcium; (4) Hydrogen; and (5) Bicarbonate. Since hydrogen and
bicarbonate ions are primarily involved in pH balance, their discussion will be delayed
until that point. Sodium, potassium and calcium will be considered here. Recall from the
prior discussion of fluid composition that sodium is primarily an extracellular ion, that
potassium occurs primarily intracellularly, and that calcium performs a variety of
functions. The important role of calcium in this discussion is its control of membrane
permeability. Being a positive ion, calcium that associates with the plasma membrane
and serves to repel other positive ions (like charge repulsions) to help control the
membrane’s permeability to positive ions.
Recall from your background study of excitable cells, such as nerve and muscle,
that sodium and potassium are essential for the action potentials conducted along the
surface membranes of these cell types. Sodium, an extracellular ion, enters an excitable
cell during the depolarization phase of the action potential. Whereas potassium, an
intracellular ion, leaves an excitable cell during the repolarization phase of the action
potential. All cells, including excitable cells in the “resting” state (not conducting an
action potential), are polarized such that the interior is negative relative to the exterior.
This difference in net charge across a cell’s membrane is due in part to the intracellular
proteins and body pH. Since normal body pH is slightly alkaline (7.4), proteins lose
hydrogen ions (behave like acids) and show a net negative charge. Since most proteins
are intracellular, when body pH is normal the positively charged intracellular potassium
ions are offset by the negatively charged intracellular proteins to give the cell’s interior a
net negative charge compared to the outside where proteins are deficient.
REVIEW QUESTIONS:
1. How do excitable cells differ from other cells of the body regarding their response
to stimulation?
2. Refamiliarize yourself with the four phases of an action potential: (1)
depolarization; (2) repolarization; (3) hyperpolarization; and (4) return to
resting state.
3. What are the ion movements associated with each phase of the action potential?
4. What would you predict might occur with an excitable cell if the normal locations
of sodium and potassium were to be reversed?
Electrolyte Disorders / Assessment
People who have disturbances in either sodium, potassium, or calcium are
probably going to show signs and symptoms of these disturbances in organ systems
whose normal functions depend upon action potentials, particularly neuromuscular
systems. Consequently, clients showing lethargy and muscle weakness or those with
increased irritability may have an electrolyte imbalance. The assessment problem then
becomes one of identifying the electrolyte(s) involved and whether they are abnormally
high (hyper-) or low (hypo-).
Electrolyte imbalances may be “primary” or “seconday” in origin. A primary
electrolyte imbalance usually affects only one electrolyte and typically involves an
abnormality in either the intake or output of the ion of interest. For example, a high salt
diet can result in hypernatremia while some diuretics “waste” potassium and can cause
hypokalemia. Since electrolytes are assayed in blood samples, you are seeing the
intravascular concentrations expressed on a lab report. Because of capillary pore
permeability, these changes probably also appear in the interstitial fluid as well and can,
in some instances, affect intracellular concentrations. Consequently, you must know
where each of the three electrolytes occur normally so you can predict the causes and
effects of their changing concentrations. Since electrolytes are osmotically active, they
can cause fluid shifts as well.
REVIEW QUESTIONS:
1. Why would hyperkalemia cause cardiac arrest?
2. How does an EKG change with increasing potassium concentrations?
A seconday electrolyte imbalance is one resulting from an abnormality in some
other physiological function. Secondary imbalances usually affect more than one
electrolyte and are common with fluid imbalances since they occur as a result of
concentration or dilution of body fluids. For example, in renal failure the kidneys fail to
output urine and can result in increased concentrations of all electrolytes normally
excreted in the urine. Secondary imbalances are usually detected by looking first for
fluid imbalances. Skin turgor (“tenting” in dehydration and “pitting” in edema) is a good
fluid balance indicator as are sudden weight changes, blood pressure abnormalities, and
peripheral or pulmonary edema. Fluid imbalances often appear on lab reports as changes
in hematocrit (percentage of formed elements in blood). A high hematocrit suggests a
possible concentration of blood maybe due to dehydration while a low hematocrit can
appear with fluid overloads.
The table below covers electrolyte imbalances that involve Na, K, and Ca. Work
through the table as you did in previous exercises being certain that you can explain the
basic physiology underlying each cause and each clinical manifestation appearing in the
table.
You should take each one of the imbalances and be sure that you can explain the
underlying physiological abnormality underlying each cause and each clinical
manifestation. This is an excellent review of basic physiology!!
ELECTROLYTE IMBALANCES
SODIUM IMBALANCES (PRIMARY AND SECONDARY)
CONDITION CAUSE CLINICAL MANIFESTATIONS
Hyponatremia Decreased intake and adrenal
insufficiency (10); inappropriate
ADH; diaphoresis with water
replacement; diuretic therapy
Cellular swelling with cerebral edema
leading to headache, stupor and coma;
muscle weakness; decreased thirst; edema
if secondary to hypervolemia;
Hypernatremia Increased intake or renal failure Cellular shrinking with increased CNS
(10); water deprivation; decreased
ADH secretion; increased
aldosterone; liver failure;
hypothalamic lesion
irritability; increased thirst; hypotension
with oliguria if secondary to hypovolemia
POTASSIUM IMBALANCES (PRIMARY AND SECONDARY)
CONDITION CAUSE CLINICAL MANIFESTATIONS
Hypokalemia Decreased intake, adrenal cortex
hyperfunction and diuretic
therapy (10); alkalosis;
vomiting/gastric suction
Cardiac arhythmia (lower T and
appearance of U wave due to slow
repolarization) and muscle weakness;
Hyperkalemia Increased intake or renal failure
and hypoaldosteronism (10);
acidosis; RBC hemolysis;
Cardiac depression (shallow, wide QRS
with elevated T due to exaggerated
repolarization); paresthesia and/or
paralysis
CALCIUM IMBALANCES (PRIMARY AND SECONDARY)
CONDITION CAUSE CLINICAL MANIFESTATIONS
Hypocalcemia Decreased intake (10), vit. D
deficiency, hypoparathyroid;
hypoalbuminemia; alcohol abuse
or liver failure
Increased neuromuscular activity
(possible convulsions); skeletal muscle
tetany;
Hypercalcemia Increased intake (10); immobility;
hyperparathyroidism; bone
malignancies; renal failure
Decreased neuromuscular activity (stupor
to coma); renal calculi; increased fracture
risk
DISCUSSION QUESTIONS: (Post answers to the “Patho Discussion Group”)
1. Explain why hyponatremia could cause cerebral edema.
2. How could it be treated in order to get the most rapid results?

INTRODUCTION

To maintain good health, a balance of fluids and electrolytes, acids and bases must
be normally regulated for metabolic processes to be in working state.

A cell, together with its environment in any part of the body, is primarily composed
of FLUID.

Thus fluid and electrolyte balance must be maintained to promote normal function.
Potential and actual problems of fluid and electrolytes happen in all health care
settings, in every disorder and with a variety of changes that affect homeostasis.

The nurse therefore needs to FULLY understand the physiology and pathophysiology
of fluid and electrolyte alterations so as to identify or anticipate and intervene
appropriately.

Fluids

a solution of solvent and solute

Solvent

a liquid substance where particles can be dissolved

Solute

a substance, either dissolved or suspended in a solution

Solution

a homogeneous mixture of 2 or more substances of dissimilar molecular structure

usually applied to solids in liquids but applies equally to gasses in liquids

Body FluidsA.Function

1.Transporter of nutrients , wastes, hormones, proteins and etc2.Medium or milieu


for metabolic processes3.Body temperature regulation4.Lubricant of
musculoskeletal joints5.Insulator and shock absorber

B.Body Fluid Compartments

IntracellularExtracellularTranscellular

Within CellsOutside cellsContained in body cavities55% or 2/3 TBW

42.5% or 1/3 TBW2.5%Transport system of our bodyNot readily utilized by the


bodyPotassium* Phosphates Magnesium

Sodium*BicarbonatesChloride

CSF, Pleural fluid, Synovial Fluid and peritoneal fluidSecreted by epithelial cells

InterstitialIntravascularBound

Fluid surrounding the cells

Within the blood vessels20%TBW or 2/3 of ECF

1/3 of ECF Plasma 7.5%Higher protein content

Bone and Cartilage 7.5%Dense Connective tissues 7.5%

C.Body Compartment Volumes


Normal valuesPrematureTerm25 yrs45 yrs65 yrsTBW Male: Female:

80%75%60%50%

55%47%

50%45%

ICF35%35%40%Blood Volume90-100 ml/kg85 ml/kg70 ml/kg

neonates reach adult values by 2 yrs and are about half-way by 3 months

average values ~ 70 ml/100g of lean body mass

percentage of water varies with tissue type,A.lean tissues ~ 60-80%B.bone ~ 20-


25%C.fat ~ 10-15%

D.Tonicity of Body Fluids

Tonicity refers to the concentration of particles in a solution

The normal tonicity or osmolarity of body fluids is 250-300 mOsm/L1.Isotonic

Same as plasma2.Hypotonic

have a lesser or lowers solute concentration than plasma3.Hypertonic

higher or greater concentration of solutes

Common Intravenous Solutions

SolutionNaCl-K+CaGluOsm.pHLactkJ/l

D5W

000027825350840
NaCl 0.9%

1501500003005.700

NaCl 3.0%

5135130008555.700

D4W/NaCL 0.18%

3030002222823.5 – 5-5

0672

Hartmans

1291095002746.72837.8

Plasmalyte

1409852945.52784

Haemaccel

1451455.16.2502937.300

Mannitol20%

000001086.200

Dextran 70

1541540003004-700Osmole

the weight in grams of a substance producing an osmotic pressure of 22.4 atm.


when dissolved in 1.0 litre of solution

(gram molecular weight) / (no. of freely moving particles per molecule)

Osmolality

the number of osmoles of solute per kilogram of solvent

Osmolarity


the number of osmoles of solute per litre of solution

Mole

that number of molecules contained in 0.012 kg of C12, or,

the molecular weight of a substance in grams =

Avogadro's number

= 6.023 x 1023

Molality

the number of moles of solute per kilogram of solvent

Molarity

is the number of moles of solute per litre of solution

THE

Normal

DYNAMICS OF BODY FLUIDS

The methods by which electrolytes and other solutes move across biologic
membranes are Osmosis, Diffusion, Filtration and Active Transport. Osmosis,
diffusion and filtration are passive processes, while Active transport is an active
process.

1.OSMOSIS

This is the movement of water/liquid/solvent across a semi-permeable membrane


from a lesser concentration to a higher concentration

Osmotic pressure is the power of a solution to draw water across a semi-permeable


membrane

Colloid osmotic pressure (also called oncotic pressure) is the osmotic pull exerted
by plasma proteins

TE DISTURBANCES

MANIFESTATIONS OF DISORDERED WATER, ELECTROLYTE AND ACID-BASE


STATUSPrimary disturbanceAltered physiologyClinical effect

SodiumECF volumeCirculatory changesWaterECF osmolalityCerebral


changesPotassiumAction potentialNeuromuscular weakness, cardiac
effectsHydrogen ionAcid-base Balance (pH)Altered tissue function, respiratory
compensationMagnesiumCell membrane StabilityNeuromuscular, vascular and
cardiac effects

Phosphate

Cellular energeticWidespread tissue effects

1)

What percentage of body weight in men is water?a.15b.30

c.

60T [ About 50% of body weight in women and 60% in men is water. Body fat
content influences the proportion of body weight that is water. As body fat
increases, water declines as a proportion of body weight. As muscle mass increases,
water increases as a proportion of body weight. Intracellular fluid (ICF) is the largest
compartment and is 60% of total body weight. Extracellular fluid (ECF) is 40% of
TBW. The solutes in the ICF and ECF compartments are different. See figure below.
In ICF the main cation is potassium and the anions are phosphates and bicarbonate.
In ECF the main cation is sodium, and chloride and bicarbonate are the anions.]

d.90

2)

What percentage of total body water is intracellular fluid?

a.30
b.40

c.60T [ 55 to 75% of total body water is intracellular and 25 to 45% is extracellular.


The total body water is approximately 40 liters. About 25 liters is inside cells (ICF
compartment). Some 15 liters is in the extracellular fluid (ECF) compartment. The
ECF is further subdivided into intravascular and extravascular spaces in a ratio of
1:3. Of the ECF, the plasma is only 3 liters. 12 liters is interstitial fluid outside the
cells. See figure below.]

d.80

3)Plasma volume is ------------- L

a.

3T [ 25% of the ECF is intravascular subcompartment. This corresponds to 3 - 3.5 L


of plasma. The remaining 75% of ECF volume is in the interstitial spaces. This
corresponds to 10 – 12 L.]b.6c.9

d.

12

4)

If 1 L of solute-free water is lost from the body, how much fluid is lost by the ICF
compartment?a.333 mL

b.

667 mL T [ Water is distributed between the ICF and the ECF in a 2:1 ratio.
Therefore, a given amount of solute-free water loss will result in a twofold greater
reduction in the ICF compartment than the ECF compartment. If 1 L of water is lost,
the ICF volume will decrease by 667 mL, whereas the ECF volume will fall by only
333 mL. If the 1L of fluid lost is isoosmotic, ECF compartment will decrease by 1 L
because Na+ is largely restricted to the ECF.]c.1 Ld.None

5)

Extracellular osmolality in a healthy adult

a.

More than intracellular osmolality

b.

Less than intracellular osmolality


c.

Same as intracellular osmolality[ Osmolality is a measure of the total number of


solutes per mass of water. Osmolality is the solute or particle concentration of a
fluid. It is expressed as milliosmoles per kilogram of water (mosmol/kg H

O). The normal plasma osmolality is 275 to 290 mosmol/kg. The extracellular and
intracellular solutes (or osmoles) are markedly different, but water crosses cell
membranes to achieve osmotic equilibrium. Water moves across cell membranes
and distributes between ICF and ECF until the osmolality in these two compartments
is the same. Therefore, ECF osmolality is equal to ICF osmolality.]

d.

Any of the above depending on fluid intake

6)

ECF osmole(s)a.Na+

b.

Cl-c.HCO3-

d.

All of the aboveT [ Sodium is the predominant cation in ECF and associates with the
anions chloride and bicarbonate. These three electrolytes account for more than
90% of the active osmoles in ECF. The predominant cation in ICF is potassium. K+ is
electrochemically balanced primarily by organic phosphates. In addition, DNA, RNA,
and phosphate esters (ATP, creatine phosphate, and phospholipids) are anionic and
provide a negative charge to balance the positive charge of potassium in
intracellular water (ICF). K+ and phosphate esters are the predominant ICF
osmoles. Solutes that are restricted to the ECF or the ICF determine the effective
osmolality (or tonicity) of that compartment. Na+ is largely restricted to the
extracellular compartment. Therefore, total body Na+ content is a reflection of ECF
volume. Hyponatremia or hypernatremia is due to disorders of water homeostasis.]

7)

ICF osmole(s)a.ATP

b.

Creatine phosphate

c.
K+d.Phospholipids

e.

All of the aboveT [ K+ is predominantly limited to the ICF. The major intracellular
anions are phosphates and negatively charged proteins. These are necessary for
normal cell function. Therefore, the number of intracellular particles is relatively
constant. Therefore, any change in ICF osmolality is usually due to a change in ICF
water content.]

8)

The main extracellular cation

a.

Potassium[ Potassium is the dominant cation in the ICF.] b.Calciumc.Magnesium

d.

SodiumT [ Sodium is the dominant extracellular cation. Chloride and bicarbonate


are the dominant ECF anions. A typical diet contains more sodium than daily
requirements. Therefore, dietary intake of Na+ results in ECF volume expansion.
This in turn promotes increased renal Na+ excretion to maintain Na+ balance. The
amount of sodium excreted is equal to the amount ingested per day.]

e.

Albumin[ An important difference between the plasma and interstitial


compartments of the ECF is that only plasma contains significant concentrations of
protein.]

9)

What maintains the difference in cation concentration between the ICF and ECF?

a.

Sodium-potassium pumpT [ Themajor force maintaining the difference in cation


concentration between the ICF and ECF is the sodium-potassium pump (Na,K-
activated ATPase) See figure below. Na

,K

+
-ATPase moves three sodium molecules out of the cell while concurrently two
potassium ions in ECF enter the cell. With three cations transported out and two
cations transported into the cell, the consequence of Na

,K

-ATPase activity is a net negative intracellular charge.]b.RMPc.Osmotic


pressured.Intracellular proteins

10)

What maintains the difference in cation concentration between the ICF and ECF?

a.

Na

,K

-adenosine triphosphate

b.

Cell membrane sodium conductance pathways

c.

Cell membrane potassium conductance pathways

d.

Free movement of water

e.

All of the aboveT [ Sodium is the major cation in the ECF. Chloride and bicarbonate
are the major accompanying anions in the ECF. Potassium is the major cation in the
ICF. Negative charges on organic molecules maintain electroneutrality with
potassium in the ICF. The difference in cationic solute composition between these
two compartments is maintained by the activity of Na

+
,K

-adenosine triphosphate (ATPase) operating in concert with cell membrane sodium


and potassium conductance pathways. The free movement of water ensures that
the sodium concentration in ECF is nearly equivalent to the potassium concentration
in ICF.]

11)

What regulates Na+ balance

a.

Sodium intake and thirst[ Na+ balance is regulated by varying Na+ excretion.]

b.

GFR[ Na+ excretion is mainly regulated by tubular Na+ reabsorption (not by GFR).]

c.

Sodium reabsorptionT [ About 65% of filtered Na+ is reabsorbed in the proximal


convoluted tubule. Further reabsorption (30%) occurs in the thick ascending limb of
the loop of Henle. About 5% of filtered Na+ is reabsorbed in the distal convoluted
tubule. Final Na+ reabsorption occurs in the cortical and medullary collecting ducts.
The amount of sodium excreted is equal to the amount ingested per day.]

d.

AVP [ Water metabolism is controlled primarily by arginine vasopressin. Sodium


metabolism is predominately regulated by the renin-angiotensin-aldosterone
system.]

12)

Osmotic adaptation occurs ina.Placenta during delivery

b.

Glomeruli during water load

c.

Normal brain T [ Brain cells can vary the number of intracellular solutes to protect
against large water shifts and resultant change in intraneuronal osmolality. This
process is called osmotic adaptation and occurs in chronic hyponatremia and
hypernatremia. Osmotic adaptation is mediated initially by shifts of K+ and Na+
and later by osmolytes.]

d.

Muscles during exercise

13)

True statement(s)

a.

Osmolality refers to the concentration of all solutes

b.

Tonicity refers to the concentration of solutes that are “effective” in eliciting a water
shift between body fluid compartments

c.

Urea is not an effective solute [ Osmolality refers to the concentration of all solutes.
Tonicity refers to the concentration of solutes that are “effective” in eliciting a water
shift between body fluid compartments. Addition or removal of solutes causes shift
of water to restore the equality of solute concentrations. Therefore, they are
considered “effective solutes”. Solutes such as urea do not elicit such a sustained
shift in water. Therefore, urea is not considered effective solute, although they
contribute to the laboratory measurement of fluid osmolality.]

d.

Addition of water without solutes results in reduction in both osmolality and tonicity
[ The addition of water without solutes results in reduction in both osmolality and
tonicity of all body fluid compartments. The removal of water without solutes results
in increase in both osmolality and tonicity of all body fluid compartments.]

e.

All of the aboveT [ All are false statements.]

14)

Ineffective osmole

a.

Sodium[ The ECF volume is a reflection of total body Na+ content. Na+ excess or
deficit are manifest as edematous or hypovolemic states, respectively.]
b.Potassium

c.

UreaT [ Urea and glucose do not contribute to water shift across cell membranes.
Therefore, they are known as ineffective osmoles.]

d.

Osmolytes [ Osmolytes are organic solutes (e.g., inositol, betaine, and glutamine).]

15)

Which of the following is the least important source of obligate water loss?

a.

Urine[ Metabolism of a normal diet generates about 600 mosmol/d. Therefore, 600
mosmols must be excreted per day through urine, primarily as urea and
electrolytes. The maximal urine osmolality that can be achieved is 1200 mosmol/kg.
Thus, a minimum urine output of 500 mL daily is required for excreting the daily
solute load. Oliguria is urine output < 500 mL/day. Water intake must equal water
excretion to maintain a steady state. Daily water intake exceeds physiologic
requirements in normal physiological conditions.]

b.

Stool T [ Gastrointestinal excretion is only a minor component of total water output.


It becomes an important route of water loss in patients with vomiting, diarrhea, or
high enterostomyoutput states.]

c.

Evaporation from the skin [ Evaporative or insensitive water losses are important in
the regulation of body temperature.]

d.

Evaporation from the respiratory tract

16)

What is the primary stimulus for drinking water?

a.

Thirst T [ The primary stimulus for drinking water is thirst. The thirst center is
situated in the organum vasculosum of the anterior hypothalamus. Thirst is caused
by an increase in effective osmolality or a decrease in ECF volume or blood
pressure. The osmotic threshold for thirst is about 295 mosmol/kg and varies among
individuals. Reduction of ECF volume also stimulates thirst by means of angiotensin
2, even when body tonicity is not elevated.]

b.

Reduction of ECF volume [ Reduction of ECF volume is a very potent stimulus for
release of AVP (carried by the ninth and tenth cranial nerves), even when body
tonicity is not elevated. Reduction of ECF volume also stimulates thirst by means of
angiotensin 2.]

c.

Hypothalamus [ Osmoreceptors are located in the supraoptic and paraventricular


nuclei of the hypothalamus. They are stimulated by a rise in tonicity. The
osmoreceptors stimulate the release of AVP from storage sites in the posterior
pituitary gland. Even very small changes in tonicity (in the range of 2%) cause
changes in release of AVP and the perception of thirst. An increase or decrease in
tonicity is sensed by hypothalamic osmoreceptors, leading to enhancement or
suppression of AVP secretion.]

d.

Glucose[ Urea and glucose are ineffective osmoles. They do not stimulate thirst.]

e.

Urea

17)

What is the primary determinant of extracellular fluid osmolality?

a.

WaterT [ Body water is the primary determinant of extracellular fluid osmolality and
disturbances in water balance primarily affect body fluid tonicity. Disorders of body
water balance can cause hypotonicity or hyperotonicity. When there is an excess of
body water relative to body solute, hypotonicity results. When there is a deficiency
of body water relative to body solute, hyperotonicity develops. The main constituent
of plasma osmolality is sodium. Therefore, hypotonic disease states are
characterized by hyponatremia and hypertonic disease states are characterized by
hypernatremia. Disturbances in sodium balance primarily affect ECF volume]
b.Glucosec.Uread.Hemoglobin

18)

What determines plasma osmolality?a.Glucoseb.Urea


c.

Sodium T [ Sodium is actively pumped out of cells by the Na+, K+-ATPase pump. As
a result, 90% of all Na+ is extracellular. The major ECF solutes are Na+ salts.
Therefore, plasma Na+ concentration determines osmolality. The normal plasma
osmolality is 275 to 290 mosmol/kg. The plasma osmolality does not vary by more
than 2%. The intake of solute-free water must be balanced by the loss of the same
volume of electrolyte-free water. Impaired free water excretion will lead to
hyponatremia.]d.Potassiume.Calcium

19)

What determines ECF volume?

a.

Sodium T [ Total body sodium is the principal determinant of ECF volume. Most of
the body's sodium is located in the ECF. The regulation of sodium excretion by the
kidney maintains normal ECF and hence plasma volume. The glomerular filtration
rate is 125 ml/min (80 liters/day) in a typical adult. Over 99% of this filtered fluid is
reabsorbed as a result of tubular reabsorption of sodium.]b.Reninc.Aldosterone

d.

Baroreceptors

20)

What is the osmotic threshold for AVP release?a.275

b.

285 mosmol/kgT [ The major stimulus for AVP secretion is hypertonicity. The
osmotic threshold for AVP release is 280 to 290 mosmol/kg. Arginine vasopressin
(AVP) is a polypeptide synthesized in the supraoptic and para-ventricular nuclei of
the hypothalamus and secreted by the posterior pituitary gland. An increase or
decrease in tonicity is sensed by hypothalamic osmoreceptors, leading to
enhancement or suppression of AVP secretion.]

c.300

d.

325

21)

What regulates water homeostasis


a.

Thirst [ Fluid homeostasis depends on proper water intake (regulated by thirst


mechanism) and on urinary excretion of free water (regulated by AVP).]b.Arginine
vasopressinc.Kidneys

d.

All of the aboveT [ The serum sodium concentration and thus serum osmolality are
controlled by water homeostasis. Water homeostasis is mediated by thirst, arginine
vasopressin, and the kidneys. Abnormal water balance manifests as an abnormality
in the serum sodium concentration (hypernatremia or hyponatremia).]

CLINICAL FEATURES OF HYPOVOLAEMIA AND HYPERVOLAEMIA

HypovolaemiaHypervolaemia SymptomsThirstDizziness on standingWeakness

Edema BreathlessnessSignsTachycardiaHypotensionDry tongue Reduced skin


turgorReduced urine outputConfusion

Peripheral edemaRaised JVPLung crepitationsPleural effusionAscitesWeight gain

HYPOVOLEMIA

CAUSES OF SODIUM AND WATER DEPLETION

MechanismExamplesInadequate intakeEnvironmental deprivation, inadequate


therapeutic replacementGastrointestinal sodium LossVomiting, diarrhea,
nasogastric suction, external fistulaSkin sodium lossExcessive sweating, burnsRenal
sodium lossDiuretic therapy, mineralocorticoid deficiency,
tubulointerstitialdiseaseInternal sequestrationBowel obstruction, peritonitis,
pancreatitis, crush injury

In internal sequestration total body sodium and water may be normal or increased.

1)

What is/are the cause(s) of hypovolemia with normal renal function?

a.

High-protein hyperalimentation[ Hypovolemia is volume depletion leading to ECF


volume contraction due to loss of both salt and water exceeding intake. Increased
renal filtration of non-reabsorbed solutes (e.g., glucose, urea) impairs tubular
reabsorption of Na+ and water, leading to an osmotic (solute) diuresis. This occurs
in poorly controlled diabetes mellitus and in patients receiving high-protein
hyperalimentation.]

b.
Mannitol IV[ The renal tubule is impermeable to mannitol. Therefore, mannitol
produces an osmotic diuresis because mannitol can be excreted along with water
only.]

c.

Hypoaldosteronism [ Mineralocorticoid deficiency (hypoaldosteronism) causes salt


wasting.]

d.

Central diabetes insipidus [ Central diabetes is due to impaired secretion of AVP.


Nephrogenic diabetes insipidus is due to renal unresponsiveness to AVP.]

e.

All of the aboveT [ Excessive renal losses of Na+ and water may also occur during
the diuretic phase of acute tubular necrosis and following the relief of bilateral
urinary tract obstruction.]History can determine the cause of hypovolemia
(bleeding, vomiting, diarrhea, polyuria, medications, diaphoresis).

2)

How much fluid enters the gastrointestinal tract daily?a.2 Lb.5 L

c.

9 LT [ About 9 L of fluid enters the GIT daily, 2 L by ingestion and 7 L by secretion.


Almost 98% of this volume is reabsorbed so that fecal fluid loss is only 100 to 200
mL/d. Impaired gastrointestinal reabsorption or increased secretion leads to volume
depletion.]d.14 L

e.

20 L

3)

Diarrhea causes

a.

Metabolic alkalosis T [ Diarrhea often causes metabolic alkalosis because biliary,


pancreatic, and intestinal secretions are alkaline (high HCO3- concentration).]

b.

Metabolic acidosis [ Gastric secretions have high H+ concentration (low pH).


Therefore, vomiting often causes metabolic acidosis.]c.Lactic acidosis
d.

Hyperkalemia 4)Sweat

a.

Usually hypertonic but may be isotonic

b.

Usually isotonic but may be hypertonic

c.

HypotonicT [ Sweat is hypotonic. Therefore, excessive sweating (e.g., febrile


illnesses, prolonged heat exposure), more loss of water than Na+, but continued
Na+ loss is manifest as hypovolaemia. The Na+ concentration of sweat is normally
20 to 50 mEq/L and decreases with profuse sweating due to the action of
aldosterone.]d.Hypertonic

5)

Third-space is in equilibrium witha.ECFb.ICFc.Both

d.

NeitherT [ Examples of “third-space” are peritoneal space, retroperitoneal space,


lumen of GIT, and subcutaneous tissue. Third space compartment is extracellular
but is not in equilibrium with either the ECF or the ICF. In pathologic conditions, ECF
can be sequestered into third-space compartments within the body without a
history of fluid loss. In such cases, the clinical manifestations are those of real
hypovolemia because the sequestered compartment is not in hemodynamic
equilibrium with the ECF. The fluid is lost from the ECF and can result in
hypovolemia. Examples include sequestration of fluid in the bowel lumen in
gastrointestinal obstruction, in the subcutaneous tissues in severe burns or trauma,
in the retroperitoneal space in acute pancreatitis, and in the peritoneal cavity in
peritonitis or malignant ascites. These extrarenal causes of absolute hypovolemia
stimulate renal sodium and fluid retention.]

6)

ECF volume contraction results in

a.

Hypotension [ Clinically ECF volume contraction manifest as a decreased plasma


volume and hypotension. Hypotension is due to decreased venous return (preload)
and diminished cardiac output.]
b.

Stimulation of baroreceptors [ Hypotension stimulates baroreceptors in the carotid


sinus and aortic arch.]

c.

Stimulation of sympathetic nervous system[ Hypotension activates the sympathetic


nervous system and the renin-angiotensin system.]

d.

Stimulation of renin secretion

e.

All of the aboveT [ The net effect is to maintain mean arterial pressure and cerebral
and coronary perfusion. In contrast to this cardiovascular response, the renal
response attempts to restore the ECF volume.]

7)

Renal response(s) to ECF volume contraction

a.

Increased GFR[ The GFR and filtered load of Na+ is decreased.]

b.

Vasodilatation of afferent arterioles[ Increased sympathetic tone decreases GFR by


causing preferential afferent arteriolar vasoconstriction.]

c.

Increased reabsorption of sodiumT [ There is increased tubular reabsorption of Na+.


Increased sympathetic tone increases proximal tubular Na+ reabsorption.]

d.

Increased atrial natriuretic peptide [ Sodium is also reabsorbed in the proximal


convoluted tubule. Enhanced reabsorption of Na+ by the collecting duct is in
response to increased secretion of angiotensin 2, aldosterone and AVP and
suppressed ANP secretion.] e.All of the above

CLINICAL FEATURES OF ECF VOLUME CONTRACTION

Symptoms (nonspecific and secondary to electrolyte imbalances and tissue


hypoperfusion)
Fatigue and weakness

Thirst

Muscle cramps

Postural dizziness

Oliguria

Confusion

Cyanosis

Signs of decreased interstitial fluid

Diminished skin turgor

Dry oral mucous membranes

Signs of intravascular volume contraction

Postural tachycardia

Postural hypotension

Decreased jugular venous pressure

Signs of hypovolemic shock

Tachycardia

Hypotension

Cold and clammy limbs

Oliguria

Altered mental status

Cyanosis

History is often helpful in determining the cause of hypovolemia (bleeding,


vomiting, diarrhea, polyuria, medications, diaphoresis).

Absence of symptoms does not exclude mild to moderate hypovolemia, especially if


the volume loss has occurred gradually.

Intravascular volume contraction of less than 5% does not usually cause symptoms
or signs. Symptoms and signs begin to appear with intravascular volume
contraction of 5 - 15%.
Severe degrees of hypovolemia (intravascular volume contraction > 15%) cause
hypotension, peripheral cyanosis, cold extremities, and reduced levels of
consciousness.

Thirst may be an early manifestation but more likely reflects a concomitant


hypertonic state.

Reduced skin turgor and dry mucous membranes are not reliable indicators of
hypovolemia.

8)

Laboratory finding(s) in loss of sodium and water

a.

Normal plasma sodium concentration[ Plasma sodium concentration may be normal


if losses of salt and water are parallel.]

b.

Raised plasma urea [ The plasma urea concentration rises because urea excretion is
reduced. Hypovolemia decreases GFR and urea excretion.]

c.

Normal plasma creatinine [ Plasma creatinine may be relatively normal early in


hypovolaemic states.]

d.

Raised plasma uric acid

e.

All of the above [ The urine specific gravity and osmolality increases because urine
concentrating mechanisms are activated to conserve water. Urine sodium
concentration falls]

9)

BUN:creatinine ratio of 20:1

a.

Normal [ Normally, the BUN:creatinine ratio is about 10:1.]

b.

Glomerulonephritis
c.

Prerenal azotemia [ In hypovolemia, the blood urea nitrogen (BUN) and plasma
creatinine concentrations tend to be elevated due to decreased GFR. In prerenal
azotemia, hypovolemia leads to increased urea reabsorption and a proportionately
greater elevation in BUN than plasma creatinine. Therefore, BUN:creatinine ratio is
20:1 or higher.]

d.

GI bleeding [ An increased BUN (relative to creatinine) may also be due to increased


urea production that occurs with hyperalimentation (high-protein), glucocorticoid
therapy, and gastrointestinal bleeding.]

e.

Both c and d are trueT

10)

In hypovolemia

a.

Low urine sodium[ Hypovolemia causes enhanced renal Na+ and water
reabsorption. Therefore, the urine Na+ concentration is usually <20 mEq/L.]

b.

High urine specific gravity[ The urine specific gravity in hypovolemic patients is
usually > 1.015 due to increased AVP secretion.]

c.

High urine osmolality[ The urine osmolality in hypovolemic patients is usually > 450
mosmol/kg due to increased AVP secretion.]

d.

All of the aboveT [ All these are the appropriate renal response to hypovolemia. In
hypovolemia due to diabetes insipidus, urine osmolality and specific gravity are
very low and indicates dilute urine. If no ADH is circulating in plasma, the osmolality
of urine produced by the kidney can be less than 100 mmol/kg H

O. If a maximum level of ADH is circulating in plasma, the osmolality of urine


produced by the kidney can exceed 1200 mmol/kg H
2

O.]

11)

A patient has quickly lost 1 L of whole blood following an accident. What features
can you expect in such a patient? [ Answer – all choices given below are true]

a.

Tachycardia

b.

Postural hypotension

c.

Peripheral vasoconstriction with cool extremities

d.

Oliguria - When fluid loss is extrarenal, there is water and sodium retention by the
kidneys. This normal renal response results in oliguria with an elevated urine
specific gravity (>1.020) and osmolality (>400 mOsm/kg), a sodium concentration
less than 20 mEq/L and a fractional excretion of sodium < 1%.

e.

Collapsed neck veins - Jugular venous pressure may fall (CVP < 5 cm H

O)

f.

Normal hemoglobin – Hemoglobin may remain constant initially. Normal hemoglobin


levels do not rule out bleeding as a cause of hypovolemia. Later, hemoglobin falls
due to movement of ECF from the interstitial to the intravascular compartment

g.

Normal blood urea - Blood urea may also remain constant initially. Later, blood urea
may increase reduced renal blood flow and the effects of destruction of
erythrocytes in the gastrointestinal tract

h.
Normal serum sodium and potassium - Sodium and potassium concentration and
acid-base parameters are not likely to change initially

12)

How much sodium is present in 1 L of normal saline?

a.

77 mEq/L [ Half-normal saline or 0.45% NaCl has77 mEq/L of Na+. Hypernatremia


indicates a proportionally greater deficit of water than Na+. Therefore, its correction
requires a hypotonic solution such as half-normal saline or 5% dextrose in water.]

b.

154 mEq/L T [ Normal saline or 0.9% NaCl has 154 mEq/L of Na+. See table below.
This is the solution of choice in normonatremic and mildly hyponatremic individuals.
It should be administered initially in patients with hypotension or shock. Patients
with significant hemorrhage, anemia, or intravascular volume depletion may require
blood transfusion or colloid-containing solutions (albumin, dextran).]

c.

513 mEq/L [ Severe hyponatremia may require hypertonic saline (3.0% NaCl or 513
mEq/L Na+).

d.

616 mEq/L

COMPOSITION OF ISOTONIC IV FLUIDS (1 L)FluidGlucoseCaloriesNa

Cl

Other

5% dextrose

50 g200000

Normal saline

001541540

13)
What is the increase in plasma volume if 1 L of isotonic saline is given IV?a.1000 mL

b.

600 mL

c.

300 mLT [ Retention of 1 L of infused isotonic saline increases plasma volume by


about 300 mL. The remaining portion is distributed in the interstitial
subcompartment of the ECF.]

d.

150mL[ In contrast, a solution of 5% dextrose in water (D

W) is equivalent to administering solute-free water. It distributes uniformly


throughout all body fluid compartments. A third of the retained volume of 5%
dextrose remains in the ECF compartment and only 10 to 15% remains in the
intravascular compartment). The retained solute-free volume reduces body tonicity
and the plasma

sodium concentration. 5% dextrose in water is useful when hypovolemia is


accompanied by hypertonicity and hypernatremia.]

e.

75 mL[ Infusing half isotonic saline (0.45% sodium chloride plus 5% glucose) is
equivalent to infusing half that volume as solute-free water (distributed throughout
body fluid compartments) and the other half as isotonic saline (confined to the ECF
compartment).]

14)

What percentage of a unit of packed red blood cells given remains in the vascular
compartment?

a.

100 [ A unit of infused packed red blood cells remains entirely in the vascular
compartment. However, erythrocytes are actually considered part of the
intracellular compartment. Packed red cells are used in the treatment of
hemorrhage to restore oxygen carriage and delivery and not as ECF volume
replacement.]b.66c.33d.15

HYPERNATREMIA
1)

Hypernatremia is plasma Na+ concentration greater than -------- mEql/La.135

b.

145T [ Hypernatremia is plasma Na+ concentration > 145 mEql/L. Patients have
moderate hypernatremia if their serum Na+ is 146 to 159 mEq/L. Patients with Na+
greater than 160 mEq/L have severe, life-threatening hypernatremia.
Hypernatremia may be due to primary Na+ gain or water deficit. Water loss is the
most common cause of acute hypernatremia. Hypernatremia can develop in
patients who do not replace the water lost after excessive sweating in a hot
environment, vomiting or diarrhea.]c.155

d.

165

CAUSES OF HYPERNATREMIA

Net water lossPure water

Unreplaced insensible losses (dermal and respiratory)

Hypodipsia

Diabetes insipidus

Hypotonic fluid

Renal causes

Diuretics

Osmotic diuresis (glucose, urea, mannitol)

Postobstructive diuresis

Polyuric phase of acute tubular necrosis

Intrinsic renal disease


Gastrointestinal causes

Vomiting

Diarrhea

Nasogastric drainage

Cutaneous causes

Burns

Excessive sweating

Hypertonic sodium gain

Hypertonic sodium bicarbonate or sodium chloride infusion

Hypertonic feeding preparation

Ingestion of sodium chloride (e.g., sea water)

Hypertonic saline enemas

Intrauterine injection of hypertonic saline

Hypertonic dialysis

Primary hyperaldosteronism

Cushing’s syndrome

2)

Hypernatremia - True statement

a.

Hyperosmolar sate [ Na+ and its accompanying anions are the major effective ECF
osmoles. Therefore, hypernatremia is a state of hyperosmolality and results in ICF
volume contraction. Sodium is impermeable. It contributes to tonicity and induces
the movement of water across cell membranes. Therefore, hypernatremia invariably
denotes hypertonic hyperosmolality and always causes cellular dehydration, at least
transiently.]

b.

Results in ICF volume contraction[ Hypernatremia causes hypertonicity of ECF.


Therefore, water shifts out of cells, leading to a contracted ICF volume. Neurologic
damage as a result of contraction of brain cell volume is the primary risk associated
with hypernatremia.]

c.

Stimulates thirst [ Hypernatremia stimulates thirst and thus increases water intake.
The severity of hyperosmolality is typically mild unless the thirst mechanism is
abnormal or access to water is limited (infants, postoperative state, impaired
mental status, and intubated patients in the intensive care unit.]

d.

Stimulates AVP secretion[ Another response to hypernatremia is excretion of a


minimum volume of maximally concentrated urine. This is due to increased AVP
secretion in response to hypertonia.]

e.

All of the aboveT [ The first question to be answered in any patient with
hypernatremia is why there has been inadequate intake of water.

Hypernatremia is rare in conscious patients who have free access to water because
of the extreme sensitivity of the thirst mechanism.]

3)

If 1 L of water is lost from the body, how much fluid is lost by the ICF compartment?
a.333 mL

b.

667 mLT [ Water is distributed between the ICF and the ECF in a 2:1 ratio.
Therefore, a given amount of solute-free water loss will result in a twofold greater
reduction in the ICF compartment than the ECF compartment. If 1 L of water is lost,
the ICF volume will decrease by 667 mL, whereas the ECF volume will fall by only
333 mL. If the 1L of fluid lost is isoosmotic, ECF compartment will decrease by 1 L
because Na+ is largely restricted to the ECF.]c.1 Ld.None

4)
What is the most common cause of hypernatremia?

a.

Gain of sodium[ A primary Na+ gain is an uncommon cause of hypernatremia. For


example, inadvertent administration of hypertonic NaCl or replacing sugar with salt
in infant formula can produce this complication.]

b.

Hyperemesis gravidarum[ Pregnant women, in the second or third trimester, may


develop nephrogenic diabetes insipidus due to excessive elaboration of
vasopressinase by the placenta.]

a.

Loss of waterT [ Most cases of hypernatremia are due to loss of free water.
Hypernatremia with hypovolaemia results from fluid losses that is more than
sodium loss. Renal water loss (e.g., diuretics) is the most common cause of
hypernatremia. Diarrhea is the most common gastrointestinal cause of
hypernatremia. Profuse sweating is also a major cause of fluid loss resulting in
hypernatremia with hypovolaemia.]

b.

Renal failure

c.

Primary hypodipsia[ Primary hypodipsia results from damage to the hypothalamic


osmoreceptors that control thirst. Thirst is impaired. Primary hypodipsia may be due
to granulomatous disease, vascular occlusion, or tumors.]

5)

Which of the following is a cause of diarrhea with hyponatremia?

a.

Lactulose [ Osmotic diarrheas (induced by lactulose, sorbitol, or malabsorption of


carbohydrate) and viral gastroenteritides cause more water loss than Na+ and K+
loss.]b.Sorbitolc.Malabsorption

d.

Viral gastroenteritides

e.
CholeraT [ Secretory diarrheas (e.g., cholera, carcinoid, VIPoma) have a fecal
osmolality similar to that of plasma. Diarrhea due these present with ECF volume
contraction and a normal plasma Na+ concentration or hyponatremia.]

6)

Fecal osmolality

a.

Equal to the sum of stool concentrations of Na+ and K+

b.

Half the sum of stool concentrations of Na+ and K+

c.

Twice the sum of stool concentrations of Na+ and K+T [ The stool osmolality is
assumed to be 300 mosmol/kg H

O. When the calculated difference is > 50, an osmotic gap is present. This suggests
that the diarrhea is due to a nonabsorbed dietary nutrient, e.g., a fatty acid and/or
carbohydrate. When this difference is < 25, it is presumed that a dietary nutrient is
not responsible for the diarrhea.]

d.

Thrice the sum of the stool concentrations of Na+ and K+

7)

Insensible losses are increased witha.Feverb.Exercise

c.

Severe burns

d.

Mechanical ventilation

e.

All of the aboveT [ Insensible loss of water is due to evaporation from the skin and
respiratory tract. Insensible losses are increased with fever, exercise, heat
exposure, and severe burns and in mechanically ventilated patients. The Na+
concentration of sweat decreases with profuse perspiration, thereby increasing
solute-free water loss.]

8)

What is the most common cause of hypernatremia?

a.

Renal water lossT [ Renal water loss is the most common cause of hypernatremia.
The normal renal response to hypernatremia is for the nephron to generate
hyperosmolar urine and retain water. However, renal correction of hypernatremia
depends on the patient having access to water. Severe hypernatremia rarely occurs
in conscious patients because intense thirst compels them to drink water. In
contrast, severe hypernatremia can develop in sedated patients, disoriented
patients, or patients in delirium tremens.]

b.

Insensible lossc.Rota virus

d.

Diabetes insipidus

9)

Cause(s) of hypervolemic hypernatremia

a.

Administration of hypertonic sodium bicarbonate [ Hypernatremia with


hypervolemia can be caused by iatrogenic administration of hypertonic sodium
chloride or hypertonic sodium bicarbonate.]b.Administration of hypertonic sodium
chloride

c.

Mineralocorticoid excess [ Mineralocorticoid excess is suggested by the presence of


hypertension and hypokalemic metabolic alkalosis. Urine sodium concentration will
vary according to dietary intake.]

d.

All of the aboveT

10)

What is the most common cause of osmotic diuresis?


a.

Diabetes insipidus[ Failure to synthesize and release ADH or failure of the renal
tubular cells to respond to ADH can result in hypernatremia.

Hypernatremia can develop in patients with diabetes insipidus who have sustained
a large water loss. Diabetes insipidus causes nonosmotic urinary water loss and
hypernatremia. Total solute excretion must equal solute production. Persons eating
a normal diet generate about 700 mosmol/d of solutes. Therefore, daily solute
excretion in excess of 750 mosmol is an osmotic diuresis. This can be confirmed by
measuring the urine glucose and urea.]

b.

Diabetes mellitusT [ Osmotic diuresis is water loss in excess of Na+ and K+. The
most common cause of an osmotic diuresis is hyperglycemia and glucosuria in
poorly controlled diabetes mellitus.]

c.

IV mannitol

d.

High-protein diet [ Intravenous administration of mannitol and high-protein diet


(increased production of urea) can also result in an osmotic diuresis.]

11)

The main symptom of hypernatremia is due to

a.

Cerebral edema and coning[ Hyponaremia may cause cerebral edema.]

b.

Hypertension and cardiac failure [ Volume depletion is often present in patients with
a history of excessive sweating, diarrhea, or an osmotic diuresis. History and
physical examination will often provide clues as to the underlying cause of
hypernatremia. Important points to note are the absence or presence of thirst,
diaphoresis, diarrhea, polyuria and current drugs. Look for features of ECF volume
contraction. Evaluate neurologic and mental status.]

c.

Altered mental statusT [ Hypernatremia causes hypertonicity of ECF. Therefore,


water shifts out of cells, leading to a contracted ICF volume. Brain cell volume is
decreased. Hence, the major symptoms of hypernatremia are neurologic and
include altered mental status, weakness, neuromuscular irritability, focal neurologic
deficits, and occasionally coma or seizures.]

d.

Craving for ice-cold water [ Patients with polydipsia from central diabetes insipidus
tend to prefer ice-cold water.

12)

What is the feature of hypernatremia due to primary Na+ excess?

a.

ECF volume expansion and urine Na+ concentration < 40 mEq/L

b.

ECF volume expansion and urine Na+ concentration >100 mmol/LT [ Measure urine
volume and osmolality in the evaluation of hyperosmolality. ECF volume expansion
and natriuresis (urine Na+ concentration >100 mEq/L) confirms a primary Na+
excess.]

c.

ECF volume contraction and urine Na+ concentration >100 mmol/L

d.

ECF volume contraction and urine Na+ concentration < 20 mEq/L

13)

What does a low volume of maximally concentrated urine indicate?

a.

Extrarenal water loss

b.

Remote renal water loss

c.

Administration of hypertonic Na+ salt solutions

d.

Any of the aboveT [ A urine specific gravity of 1.010 units or less is a dilute urine
and suggests that ADH levels are low. A urine specific gravity greater than 1.030
units suggests that the urine being produced is close to maximum osmolality. The
correct renal response to hypernatremia is the excretion of the minimum volume
(500 mL/d) of maximally concentrated urine (urine osmolality > 800 mosmol/kg).

These findings suggest extrarenal or remote renal water loss or administration of


hypertonic Na+ salt solutions.]

14)

What is the solute excretion rate?

a.

Urine volume ÷ osmolality

b.

Urine volume X osmolalityT [ The solute excretion rate is the product of the urine
volume and osmolality.]c.Urine sodium + potassium + chloride

d.

{ Urine sodium + potassium + chloride + glucose } ÷ urine osmolalitye.{ Urine


sodium + potassium + chloride + glucose } ÷ urine volume

15)

Treatment of hypernatremiaa.Loop diureticb.Desmopressin

c.

Correct the water deficit [ The most common cause of hypernatremia is loss of
water. Treatment of patients with hypernatremia secondary to dehydration is IV or
oral administration of water. The amount of water required to correct the deficit can
be calculated from the equation given below. Hypernatremic patients typically have
reduced blood volumes. Treat these patients first with the IV infusion of isotonic
saline solutions until the contracted ECF has been restored. Then give sufficient
electrolyte-free water to enable their renal function to produce concentrated urine
and correct the hypernatremia. In patients with prolonged hyperosmolality,
aggressive treatment with hypotonic fluids may cause cerebral edema, which can
lead to coma, convulsions, and death. Lower Na+ only at a rate < 8 mEq/day. See
figure below.]d.Dialysis

e.

Tetracycline


Water deficit = plasma sodium concentration – 140

X Total body water140

Total body water is approximately 50 of lean body weight in men and and 40% of
lean body weight in women

Within minutes after the development of hypertonicity, loss of water from brain cells
causes shrinkage of the brain and an increase in osmolality. Partial restitution of
brain volume occurs within a few hours as electrolytes enter the brain cells (rapid
adaptation). The normalization of brain volume is completed within several days as
a result of the intracellular accumulation of organic osmolytes (slow adaptation).
Slow correction of the hypertonic state reestablishes normal brain osmolality
without inducing cerebral edema, as the dissipation of accumulated electrolytes and
organic osmolytes keeps pace with water repletion. In contrast, rapid correction
may result in cerebral edema as water uptake by brain cells outpaces the
dissipation of accumulated electrolytes and organic osmolytes. Such overly
aggressive therapy carriesthe risk of serious neurologic impairment due to cerebral
edema.

16)

How much is the free water deficit in a 50-kg woman with a plasma Na+
concentration of 160 mEq/L?

a.

1.9 L

b.

2.9 LT [ (20 ÷ 140) X (0.4 × 50). Rapid correction of hypernatremia can be


dangerous. A sudden decrease in osmolality may cause a rapid shift of water into
brain cells. Therefore, correct the water deficit slowly over at least 48. The safest
route of administration of water is by mouth. 5% dextrose in water or half-isotonic
saline can be given intravenously safely.]

c.

3.9 L

d.

4.9 L

17)
What is the preferred route for administering fluids in a patient with hypernatremia?

a.Intravenous through a peripheral veinb.Intravenous through a central vein

c.

Oral routeT [ The preferred route for administering fluids is the oral route or a
feeding tube. If neither is feasible, fluids should be given intravenously.]

d.

Per rectal

18)

Which fluid is least appropriate for managing hypernatremia?

a.

Pure water[ Only hypotonic fluids are appropriate, including pure water, 5 percent
dextrose, 0.2 percent sodium chloride (one-quarter isotonic saline), and 0.45
percent sodium chloride (one-half isotonic saline).

b.

5% dextrosec.0.2 percent sodium chlorided.0.45 percent sodium chloride

e.

Isotonic saline T [ Except in cases of frank circulatory compromise, 0.9 percent


sodium chloride (isotonic saline) is unsuitable for managing hypernatremia.]

nes, decreased skin turgor, fever, tachypnea, and a blood pressure of 142/82 mm
Hg without orthostatic changes. The serum sodium concentration is 168 mEq per
liter, and the body weight is 68 kg. Hypernatremia caused by pure water depletion
due to insensible losses is diagnosed, and an infusion of 5 percent dextrose is
planned. How much is the estimated volume of total body water in liters?

a.

41[ The estimated total body water (in liters) is calculated as a fraction of body
weight. The fraction is 0.6 in children; 0.6 and 0.5 in nonelderly men and women,
respectively; and 0.5 and 0.45 in elderly men and women, respectively. Normally,
extracellular and intracellular fluids account for 40 and 60 percent of total body
water, respectively. The estimated volume of total body water in a child weighing
68 kg would be 68 X 0.6 = 40.8L]

b.
34T [ 0.5 X 68]

c.

31[ The estimated volume of total body water in an elderly men and woman
weighing 68 kg would be 68 X 0.45 = 30.6 L.]

d.

25

20)

If 1 liter of 5 percent dextrose is given to the patient described above, what will be
the fall in serum sodium concentration?

a.

2.4 mEq per liter

b.

4.8 mEq per literT [ Change in serum Na

= (infusate Na

- serum Na

) ÷ (total body water + 1). According to this formula, the retention of 1 liter of 5
percent dextrose will reduce the serum sodium concentration by 4.8 mEq per liter
[ (0 – 168) ÷ (34+1) = - 4.8.]

c.

9.6 mEq per liter

d.

19.2 mEq per liter

21)

A 58-year-old woman with postoperative ileus is undergoing nasogastric suction.


She is obtunded with diminished skin turgor and mild orthostatic hypotension. The
serum sodium concentration is 158 mEq per liter, the potassium concentration is
4.0 mEq per liter, and the body weight is 63 kg. What is the treatment?

a.

0.45% sodium chloride IVT [ Hypernatremia caused by hypotonic fluid loss is the
correct diagnosis. The estimated volume of total body water is 31.5 liters (0.5 X 63).
The infusion of 1 liter of 0.45 percent sodium chloride will reduce the serum sodium
concentration by 2.5 mEq per liter (77 – 158) ÷ ( 31.5 + 1) = - 2.5. If the goal is to
reduce the serum sodium concentration by 5 mEq per liter over the next 12 hours, 2
liters of the solution is required (5÷2.5). If 1 liter is added to compensate for
ongoing losses of gastric and other fluids, a total of 3 liters will be administered for
the next 12 hours, or 250 ml per hour.]

b.

0.9% sodium chloride IV[ Although there is evidence of a depletion in the volume of
extracellular fluid, the patient’s hemodynamic status is not sufficiently
compromised to warrant the initial use of 0.9 percent sodium chloride.]

c.

Furosemide IV

d.

Furosemide oral

e.

Hemodialysis

22)

A 62-year-old man with advanced alcoholic cirrhosis is on lactulose for hepatic


encephalopathy. Examination shows confusion, ascites, and asterixis. The blood
pressure is 105/58 mm Hg in the supine position, and the pulse is 110 beats per
minute. The serum sodium concentration is 160 mEq per liter, the potassium
concentration is 2.6 mEq per liter, and the body weight is 64 kg. What is the
treatment?

a.

Increase the dose of lactulose and give IV normal saline

b.

Withdraw lactulose and give IV 0.2 percent sodium chlorideT [ The hypernatremia is
due to hypotonic sodium and potassium losses induced by lactulose therapy.
Treatment is withdrawal of lactulose and IV 0.2 percent sodium chloride containing
20 mEq of potassium chloride per liter. With the presence of ascites, the estimated
volume of total body water is about 38 liters (0.6 X 64).

c.

Furosemide IVd.Liver transplantation

e.

Potassium chloride IV

23)

A 60-year-old man has received 10 ampoules of sodium bicarbonate over six hours
during resuscitation after recurrent cardiac arrest. He is stuporous and is
undergoing mechanical ventilation. His blood pressure is 138/86 mm Hg, and
peripheral edema is present. The serum

sodium concentration is 156 mEq per liter, the body weight is 85 kg, and the urinary
output is 30 ml per hour. What is the treatment?

a.

Furosemide[ The hypernatremia is caused by hypertonic sodium gain. For its


correction, the excess sodium and water be excreted. Furosemide alone is not
enough, because furosemide- induced diuresis is equivalent to one-half isotonic
saline solution. Thus, the hypernatremia will be aggravated.]

b.

Furosemide and electrolyte-free waterT [ The correct treatment is administration of


both furosemide and electrolyte-free water. The estimated volume of total body
water is 51 liters (0.6 X85). If 1 liter of 5 percent dextrose is given, it will decrease
the serum sodium concentration by 3.0 mEq per liter (0 – 156) ÷ (51+1) = - 3.0.
Since the patient’s extracellular-fluid volume is expanded, fluids can be
administered only with great caution. Adjust fluid administration based on close
monitoring of the patient’s clinical status and serum sodium concentration.]

c.

Hemodialysis [ Hypernatremia with concurrent renal failure and volume overload is


a special problem. Diuretics cannot be relied on to reduce the expanded
extracellular-fluid volume. Therefore, hemodialysis, hemofiltration, or peritoneal
dialysis may be necessary.]

d.

Peritoneal dialysis
e.

Dopamine

24)

Not suited for correcting hypernatremia in a 50-year-old man with a serum sodium
concentration of 162 mEq per liter and a body weight of 70 kg is

a.

Isotonic salineT [ Isotonic saline is unsuitable for correcting hypernatremia.


Estimated volume of total body water is 42 liters (0.6 X 70). The retention of 1 liter
of 0.9 percent sodium chloride will decrease the serum sodium concentration by
only 0.2 mEq per liter ( 154 – 162) ÷ (42 + 1) = - 0.2). Although the sodium
concentration of the infusate is lower than the patient’s serum sodium
concentration, it is not sufficiently low to alter the hypernatremia substantially. The
only indication for administering isotonic saline to a patient with hypernatremia is a
depletion of extracellular-fluid volume that is sufficient to cause substantial
hemodynamic compromise. Even in this case, after a limited amount of isotonic
saline has been administered to stabilize the patient’s circulatory status, give a
hypotonic fluid (i.e., 0.2 percent or 0.45 percent sodium chloride). If a hypotonic
fluid is not substituted for isotonic saline, the extracellular-fluid volume may
become seriously overloaded.]b.0.2 percent sodium chloride

c.

0.45 percent sodium chloride d.All of the above

HYPONATREMIA

1)

How do disorders in sodium balance present?

a.

Altered ECF volume T [ Disorders in sodium balance present chiefly as altered ECF
volume rather than altered sodium concentration.]b.Hyponatremia

c.Hypernatremia

d.

Diabetes insipidus

2)

Hyponatremia is plasma Na less than ------------- mEq/L a.125


b.

135T [ The normal serum sodium ion concentration ranges between 135 and 145
mEq/L. Hyponatraemia is plasma Na < 135 mEq/L. Severe hyponatremia is defined
as an Na+ less than 120 mEq/L. A falling Na

indicates that the osmolality in ECF and ICF is also falling. Hypernatremia always
indicates hypertonicity, but hyponatremia can be associated with low, normal, or
high tonicity.]c.145

d.

155

CAUSES AND CLASSIFICATION OF HYPONATREMIA

High Plasma Osmolality

Hyperglycemia

Hypertonic mannitolNormal Plasma Osmolality (Pseudohyponatremia)

Hyperlipidemia

HyperparaproteinemiaLow Plasma Osmolality

High Circulating ADH

Volume depletion

Cirrhosis

Heart failure


Nephrotic syndrome

SIADH

Hypothyroidism

Hypoadrenalism (Glucocorticoid deficiency)

Drugs

Pregnancy

Severe hypoalbuminemia

Low Circulating ADH

Acute or chronic renal failure

Low solute intake (beer potomania)

Primary polydipsia

Low solute intake (beer potomania)

Primary polydipsiaPosm in mOsm/kg H

O = (2 X serum Na in mEq/L) + (glucose in mg/dL/18) + (BUN in mg/dL/2.8)


3)

What is the most common cause of hyponatremia?a.Diuretics

b.

Dilutional hyponatremiaT [ Dilutional hyponatremia is the most common form of


hyponatremia. It is most commonly caused by water retention due to impaired renal
excretion of water. With the exception of renal failure, these conditions are
characterized by high plasma concentrations of arginine vasopressin. In a minority
of cases, dilutional hyponatremia is caused by excessive water intake (e.g., primary
polydipsia). If water intake exceeds the capacity of the kidneys to excrete water,
dilution of body solutes results, causing hypo-osmolality and hypotonicity.
Hypotonicity can lead to cerebral edema.]

c.

Hyperglycemia [ Hypertonic hyponatremia is due to shift of water from cells to the


extracellular fluid. It is caused by solutes confined in the extracellular compartment
(e.g., hyperglycemia or mannitol). The serum tonicity is increased causing
dehydration of cells.]d.Pregnancy

4)

The total sodium stores of the body in dilutional hyponatremia


isa.Decreasedb.Normalc.Increased

d.

Any of the aboveT [ Hypotonic (dilutional) hyponatremia means an excess of water


in relation to existing sodium stores, which can be decreased, essentially normal, or
increased.]

5)

What is the most common cause of isotonic hyponatremia?

a.

Pseudohyponatremia T [ Isotonic hyponatremia is usually synonymous with so-


called ‘‘pseudohyponatremia’’ and must be distinguished from true hypoosmolality.
Normal serum is typically comprised of 93% water and 7% nonaqueous factors,
including lipids and proteins. The nonaqueous components usually do not affect
serum tonicity. In marked hyperproteinemia or hyperlipidemia, the nonaqueous
proportion of serum is relatively increased with respect to the aqueous portion. The
result is an artifactual decrease in the serum concentration of Na+ although the
concentration of Na+ is unchanged. Isotonic hyponatremia does not cause
movement of water between the ntracellular fluid (ICF) and extracellular fluid (ECF)
compartments.]

b.

Hyperglycemia[ Hyperglycemia

causes hypertonic hyponatremia.]

c.

High blood urea[ Unlike glucose, solutes that enter cells, such as urea or ethanol, do
not cause intracellular-to-extracellular water shift and thus do not cause
hyponatremia.]

d.

SIADH[ SIADH causes hypotonic hyponatremia.] e.All of the above

6)

What is the most common cause of hypertonic hyponatremia?

a.

Hyperglycemia T [ High osmolality indicates the presence of excessive extracellular


osmoles leading to the entry of intracellular water into the extracellular fluid and
the dilution of the sodium in the extracellular fluid. High glucose level is the
commonest cause of hypertonic hyponatremia.

The increases in plasma glucose raise serum osmolality, which pulls water out of
cells and dilutes the serum Na.

Plasma Na+ concentration falls by 1.6 mEq/L for every 100 mg/dL rise in the
plasma glucose concentration.]

b.

High blood urea[ Unlike glucose, solutes that enter cells, such as urea or ethanol, do
not cause intracellular-to-extracellular water shift and thus do not cause
hyponatremia.]

c.

Transurethral resection of the prostate[ Massive absorption of irrigant solutions that


do not contain sodium (e.g., those used during transurethral prostatectomy) can
cause severe hyponatremia. Isotonic or slightly hypotonic hyponatremia may
complicate transurethral resection of the prostate or bladder. Large volumes of
isoosmotic (mannitol) or hypoosmotic (sorbital or glycine) bladder irrigation solution
can be absorbed and result in a dilutional hyponatremia.]

d.

IV mannitol[ Hypertonic hyponatremia may also be due to intravenous


administration of mannitol.]

e.

Thiazide diuretics[ Thiazide diuretics lead to Na+ and K+ depletion and AVP-
mediated water retention.]

7)

What is the most common cause of hypotonic hyponatremia?a.Hyperglycemia

a.

Primary polydipsia[ Hypotonic hyponatremia means that water intake exceeds the
ability of the kidney to excrete water. A normal kidney can excrete 30 L of water per
day. Therefore, hyponatremia with normal renal water excretion implies that the
patient is drinking more than 30 L of water per day. This condition is referred to as
primary polydipsia. These patients should have a urine osmolality less than 100
mOsm/L. Primary polydipsia is a common condition that leads to polyuria and
polydipsia, but it is uncommon as the sole cause of hyponatremia.]b.Mannitol
infusionc.Diuretics

a.

SIADHT [ Hypotonic hyponatremia is most commonly due to decreased renal water


excretion (i.e., urine that is not adequately concentrated even when there is
hypotonia). When there is hypotonic hyponatremia, the normal response of the
kidneys is to excrete maximally diluted urine. A urine osmolality >100 mOsm/L is
inappropriate in hypotonic hyponatremia. Inappropriately concentrated urine
implies a defect in renal water excretion (failure to maximally suppress AVP levels.]

8)

Diuretic-induced hyponatremia is almost always due to

a.

ThiazidesT [ Diuretic-induced hyponatremia is almost always due to thiazide


diuretics. Thiazide diuretics lead to Na+ and K+ depletion and AVP-mediated water
retention. Thiazide diuretics reduce the reabsorption of Na+ and Cl- in the first half
of the distal convoluted tubule and a portion of the cortical ascending limb of the
loop of Henle. However, thiazides can cause excretion of a hypertonic urine and
contribute to dilutional hyponatremia.]

b.

Furosemide [ Furosemide, bumetanide, and torsemide are "loop" diuretics. They


reversibly inhibit the reabsorption of Na+, K+, and Cl- in the thick ascending limb of
Henle's loop. Hypokalemia, hyperuricemia, and hyperglycemia are observed
occasionally. The reabsorption of free H2O is decreased. Loop diuretics decrease
the tonicity of the medullary interstitium and impair maximal urinary concentrating
capacity. This limits the ability of AVP to promote water retention.]c.Bumetanide

d.

Ethracrynic acid

e.

Spiranolactone

9)

What is/are the cause(s) of hyponatremia with increased ECF volume


(hypervolemia)?

a.

Congestive heart failure [ Hypotonic hyponatremia is subdivided according to the


clinical ECF volume status. Hyponatremia with ECF volume expansion is seen in
edematous states, such as congestive heart failure, hepatic cirrhosis, and the
nephrotic syndrome. All these conditions have decreased effective circulating
arterial volume, leading to increased thirst and increased AVP levels.]

b.

Cirrhosis

c.

Nephrotic syndrome

d.

Renal failure with oliguria[ Oliguric renal failure may cause hyponatremia if water
intake is more than what the kidneys can excrete.]

e.
All of the aboveT [ The severity of hyponatremia often correlates with the severity
of the underlying condition and is an important prognostic factor. See figure below.]

10)Secondary hyperaldosteronisma.Cardiac failureb.Cirrhosis

c.

Nephrotic syndrome

d.

All of the aboveT [ These conditions are associated with sodium and fluid overload.
The reduced effective blood volume stimulates renin-angiotensin-aldosterone axis
resulting in sodium and fluid overload. See figure above.]

11)

Hypervolemic hyponatremia caused by

a.

Heart failure [ In CCF, even though there is total body ECF overload, the decreased
cardiac output causes perceived intra-arteriolar volume depletion. There is a
decrease in the effective arterial blood volume at the level of the carotid artery and
the renal afferent arteriole baroreceptors. Decreased renal perfusion activates the
RAAS, resulting in increased sodium and water reabsorption. Increased baroreceptor
firing activates non-osmotic AVP secretion, resulting in increased free water
reabsorption. These adaptive physiologic mechanisms further exacerbate
hypervolemia and progressive hyponatremia. ACE inhibitors reduce mortality in this
subgroup of patients with cardiac failure.]

b.

Cirrhosis [ Cirrhosis is the most common cause of ascites. 30% of patients with
ascites develop hyponatremia. Gastrointestinal endotoxin is less efficiently cleared
in cirrhosis due to portal-systemic shunting. This stimulates nitric oxide production
and vasodilatation. Arterial dilatation in the splanchnic vasculature, leads to arterial
underfilling and non-osmotic secretion of AVP. The increase in AVP secretion and
water retention is proportional to the severity of cirrhosis. A serum Na+ less than
125 mEq/L often indicate end-stage disease.]

c.

Nephrotic syndrome [ Patients with advanced renal disease typically develop


hyponatremia due to abnormal water retention.]

d.
All of the aboveT [ In hypervolemic hyponatremia, there is an excess in total body
water and total body sodium, resulting in edema or ascites. In many cases, the
increase in total body water is out of proportion to that of total body sodium,
causing hyponatremia. This pathophysiology occurs in congestive heart failure,
cirrhosis, and nephrotic syndrome.]

12)

Hypervolemic hyponatremia - True statement

a.

Low intravascular volume[ These patients have low ECF volume secondary to heart
failure, cirrhosis, or nephrotic syndrome. Low ECF volume stimulates nonosmotic
release of vasopressin, renal water reabsorption, and subsequent hyponatremia.]

b.

Urinary sodium is very low [ Spot urinary sodium is very low (often <10 mEq/L) due
to avid renal sodium retention.]

c.

Edema is typical[ Physical examination may show peripheral edema, ascites,


pulmonary congestion, or jugular venous distention depending on the underlying
cause.]

d.

Seen in renal failure [ Hypervolemic hyponatremia may be seen in acute or chronic


renal failure where renal excretion of sodium and water is decreased. The
hyponatremia is the result of water intake in excess of sodium intake (dilutional
hyponatremia). It is a common finding in patients with end-stage renal disease. The
U

Na

may be greater than 20 mEq/L.]

e.

All of the aboveT

CAUSES OF HYPONATRAEMIA

14)
Volume statusExamples

Hypovolaemic (sodium deficit with a relatively smaller water deficit)

Renal Na losses (e.g., thiazides)

Adrenocortical failure

Gastrointestinal Na losses (e.g., Vomiting, Diarrhoea)Euvolaemic (water retention


alone)

Primary polydipsia

Excessive electrolyte-free water infusion

SIADH

HypothyroidismHypervolaemic (sodium retention with relatively greater water


retention)

Congestive cardiac failure

Cirrhosis

Nephrotic syndrome

Chronic renal failure (during free water intake)

MOST COMMON CAUSES OF SEVERE HYPONATREMIA


Adults

Thiazides

Postoperative state

SIADH

Polydipsia in psychiatric patients

Transurethral prostatectomy Infants and children

Gastrointestinal fluid loss

Ingestion of dilute formula

Accidental ingestion of excessive water

Multiple tap-water enemas

17)

What is the most common cause of normovolemic hyponatremia?

a.

SIADHT [ Syndrome of inappropriate antidiuretic hormone secretion is the most


common cause of normovolemic hyponatremia. It is due to the nonphysiologic
release of AVP from the posterior pituitary or an ectopic source. AVP impairs renal
excretion of free water while the regulation of Na+ balance is not affected. Small-
cell lung cancer is the most common cause of SIADH. Other causes of SIADH include
stroke, meningitis, pneumonia and TB. Patients with SIADH have a serum osmolality
< 270 mmol/kg H

O along with inappropriately concentrated urine.]

b.

Hypothyroidism [ Other causes of euvolemic hyponatremia are hypothyroidism,


glucocorticoid deficiency, acute emotional stress or psychosis, certain drugs
(selective serotonin reuptake inhibitor, carbamazepine, nonsteroidal anti-
inflammatory drugs, vincristine, and

oxytocin), and primary polydipsia.]

c.
Serotonin reuptake inhibitor[ Drugs associated with euvolemic hyponatremia act
largely by stimulating release of ADH from the posterior pituitary gland.]

d.

Primary polydipsia[ Circulating ADH levels are high in all these disorders except for
primary polydipsia.]

SIADH – DIAGNOSIS

Low plasma sodium concentration (< 130 mEq/L)

Low plasma osmolality (< 270 mosmol/kg)

Urine osmolality not minimally low (> 150 mmol/kg)

Urine sodium concentration not minimally low (> 30 mEq/L)

Low-normal plasma urea, creatinine, and uric acid

No signs of hypovolemia (tachycardia, orthostatic hypotension, decreased skin


turgor, dry mucous membranes) or hypervolemia (edema, ascites)

No hypothyroidism, hypocortisolism or diuretic use

18)

Cause(s) of SIADH

a.

Hypopituitarism [ Hypopituitarism can also lead to hyponatremia. Hyponatremia is


not due to mineralocorticoid deficiency. The main regulator of aldosterone secretion
is the RAAS and not ACTH secretion by the pituitary.]

b.

Thyroid disease[Patients with primary hypothyroidism have impaired free water


excretion. Decreased free water excretion in myxedema is due to an alteration in
renal perfusion and GFR (not due to inappropriate plasma AVP elevation). The renal
changes are secondary to systemic effects of thyroid hormone deficiency on cardiac
output and peripheral vascular resistance. This can be reversed by thyroid hormone
replacement.]

c.

Adrenal diseased.All of the above

e.
None of the above T [ SIADH is a diagnosis of exclusion and can only be made in the
setting of normal renal, thyroid, and adrenal function.]

19)

What is/are the cause(s) of hyponatraemia with hypovolaemia?

a.

Diuretic therapy [ Thiazides are more commonly associated with severe


hyponatremia than are loop diuretics such as furosemide. There is high urine
sodium excretion (Na > 20 mEql/L).]b.Addison's diseasec.Vomitingd.Burns

e.

All of the aboveT [ Clinical features of hypovolaemia (tachycardia, orthostatic


hypotension, dry mucous membranes, flat neck veins, absence of edema) are
common. Urinary sodium concentration is low and plasma renin activity is
elevated.]

20)

What is/are the cause(s) of hyponatraemia with hypovolaemia due to increased


renal loss of sodium and water?

a.

Diuretic therapy [ Diuretic induced hyponatremia is more common with thiazide


diuretics than after loop diuretics. Though less potent than loop diuretics, thiazide
diuretics do not disrupt the medullary countercurrent concentrating
mechanism.]b.Mineralocorticoid deficiency

c.

Cerebral salt wasting [ Head injury and intracranial hemorrhage can induce
negative sodium balance through urinary losses. The hypovolemia induces release
of AVP. This condition is frequently difficult to distinguish from hyponatremia caused
by the SIADH.]d.Salt-losing nephropathy

e.

All of the aboveT

21)

Hypovolemic hyponatremia - True statement

a.
Low circulating ADH levels[ Patients with hypovolemic hyponatremia have a deficit
in total body sodium causing low ECF volume and nonosmotic release of antidiuretic
hormone (ADH). High circulating ADH levels stimulate renal water reabsorption and
subsequent development of hyponatremia.]

b.

Edema is typical[ Patients with hypervolemic hyponatremia have excess total body
sodium manifested by edema. Physical examination in a patient with hypovolemic
hyponatremia typically shows features of hypovolaemia (tachycardia, orthostatic
hypotension, dry mucous membranes, flat neck veins, absence of edema)]

c.

Spot urine sodium concentration is always less than 20 mEq/L if renal function is
normal[ When volume losses are nonrenal (due to hemorrhage, diarrhea, dermal
losses, or third spacing of fluids as in pancreatitis or peritonitis), the spot urine
sodium concentration is typically less than 20 mEq/L. When volume losses are due
in part to renal sodium wasting, the spot urine sodium concentration is greater than
20 mEq/L. Such renal sodium wasting may be seen in the presence of active
diuretics, mineralocorticoid deficiency, osmotic diuresis, salt-losing nephropathy,
bicarbonaturia (most commonly from vomiting), and ketonuria.]d.All of the above

e.

None of the aboveT [ All are false statements.]

22)

What is the best index of effective arterial volume?

a.

Pulse and blood pressureT [ On physical examination, the best index of effective
arterial volume is the pulse and blood pressure. The earliest clinical evidence of
decreasing blood volume is tachycardia and postural fall in BP.]

b.

Low urinary sodium

c.

Low urinary chloride d.Low fractional excretions of sodium and chloride in the urine

a.

Blood urea [ BUN is very sensitive to effective arterial volume. In patients with
normal serum creatinine, a high BUN suggests a low effective arterial volume and a
low BUN suggests a high effective arterial volume. The plasma uric acid can also be
used as a sensitive index of effective arterial volume.]

23)

Cause(s) of hyponatremia

a.

Adrenal insufficiency[ Cortisol deficiency that leads to hypersecretion of AVP. AVP is


cosecreted with corticotropin-releasing factor. Volume depletion also causes
hypersecretion of AVP. Decreased mineralocorticoids of adrenal insufficiency may
contribute to the hyponatremia.]

b.

Hypothyroidism [ Decreased cardiac output and GFR seen in hypothyroidism lead to


increased AVP secretion.]c.SIADH

d.

Primary polydipsia[ The normal kidneys can excrete about 12 L of water daily. In
psychogenic or primary polydipsia, compulsive water consumption may be much
more than this. These patients often have psychiatric illnesses.

e.

All of the aboveT [ Metabolism of a normal diet generates about 600 mosmol/d. The
minimum urine osmolality is 50 mosmol/kg. Therefore, the maximum daily urine
output will be about 12 L (600 ÷ 50 = 12).]

URINE Na+ AND OSMOLALITY IN HYPONATRAEMIA

Urine Na (mEq/L)Urine osmolality (mmol/kg)Diagnoses

Low (< 30)Low (< 100)Primary polydipsiaMalnutritionBeer excessLowHigh (>


150)Salt depletionHypovolaemiaHigh (> 40)LowDiuretic action (acute
phase)HighHighSIADHCerebral salt-wastingAdrenal insufficiency

24)

What is the diagnosis in a patient with high urine sodium and low specific gravity?

a.

SIADH

b.

Adrenal insufficiency
c.

Diuretic Td.Cardiac failure

e.

Hypovolaemia

25)

What is potomania?a.Fear of small potsb.Fear of being photographedc.Fear of


cameras

d.

Hyponatremia seen in beer drinkersT [ “Beer-drinker’s potomania” is caused by a


high in fluid (beer) intake that is low in solute. Consumption of large volumes of
beer may exceed the renal excretory capacity and result in hyponatremia. This
phenomenon is referred to as beer potomania.]e.Seen in HIV infection

26)

Hyponatremia - True statement

a.

Common abnormality [ Hyponatremia is a common electrolyte abnormality, often


detected asymptomatically.]

b.

Often asymptomatic[ Patients with mild to moderate hyponatremia (Na+ >120


mEq/L) rarely have signs or symptoms of their hyponatremia. These patients are
identified on laboratory testing.]

c.

Can cause severe cerebral dysfunction[ Cerebral symptoms are anorexia, nausea,
vomiting, confusion, lethargy, seizures and coma. Brain cells adapt to hyponatremia
by losing cellular potassium, organic solutes, and then other organic osmolytes. This
adaptation requires 48 to 72 hours and is very effective in reducing brain swelling.
Thus, when hyponatremia occurs slowly, patients have few or no symptoms. When
hyponatremia develops in less than 48 hours, adaptation has not had time to occur.
These patients are at high risk for developing cerebral edema and intracranial
hypertension. Intracranial cell swelling in patients with acute severe hyponatremia
causes headaches and confusion that may rapidly progress to coma or seizures.]

d.
Clinical features depends on the rate of development of hyponatremia [ Cerebral
symptoms depends more on how fast hyponatremia develops than on the severity
of hyponatremia. This is because when the plasma osmolality falls rapidly, water
flows into cerebral cells which become swollen and ischaemic. When hyponatraemia
develops gradually, cerebral neurons respond by reducing the intracellular
osmolality. Cerebral neurons reduce intracellular osmolality by reducing its
potassium concentration and synthesis of intracellular organic osmolytes.]

e.

All of the aboveT [ See figures below.]

27)

Symptoms of hyponatremia are predominantly

a.Cardiovascularb.Respiratory

c.

Neurologic T [ The severity of symptoms depends mainly on the rapidity of the


decrease in serum Na+. Most patients are not symptomatic until the serum Na+
decreases to less than 125 mEq/L. Symptoms are predominantly neurologic
(nausea, vomiting, headache, fatigue, irritability, and disorientation). See table
below. Severe hyponatremia can progress to seizures, brainstem herniation, and
death.]d.Renal

SYMPTOMS OF HYPONATREMIA

28)

What is the most important clinical manifestation of hyponatremia?

a.

Cerebral edemaT [ Hyponatremia leads to increased ICF volume, specifically brain


cell swelling. The symptoms of hyponatremia are primarily neurologic. Their
severity dependents on the rapidity of onset and absolute decrease in plasma Na+
concentration. Patients may be asymptomatic at first. As the plasma Na+
concentration falls, headache, confusion, and obtundation develop. Seizures and
coma do not usually occur unless the plasma Na+ concentration falls acutely below
120 mmol/L or decreases rapidly. Acute or severe hyponatremia (plasma Na+
concentration <115 mEql/L) present with altered mental status and/or
seizures.]b.Cardiac failurec.Muscle weaknessd.ECG changese.Thirst

29)

What is the most important clinical manifestation of acute hyponatremia?


a.

Intracranial hypotension

b.

Intracranial hypertension T [ Hypotonic hyponatremia causes entry of water into the


brain, resulting in cerebral edema. The surrounding cranium limits expansion of the
brain. Therefore, intracranial hypertension may develop. Brain rapidly adapts to this
change. Solutes leave brain tissues within hours, thereby inducing water loss and
ameliorating brain swelling. This process of adaptation by the brain accounts for the
relatively asymptomatic nature of even severe hyponatremia if it develops
slowly.]c.Thirst

d.

Osmotic demyelination[ Osmotic demyelination can develop one to several days


after aggressive treatment and rapid correction of hyponatremia. Shrinkage of the
brain triggers demyelination of pontine and extrapontine neurons that can cause
neurologic dysfunction, including quadriplegia, pseudobulbar palsy, seizures, coma,
and even death.]

DIAGNOSIS OF HYPONATREMIA – 4 TESTS

1.

Plasma osmolality

2.

Urine osmolality

3.

Urine Na+ concentration

4.

Urine K+ concentration

Most patients with hyponatremia have a decreased plasma osmolality. The kidneys
respond to hypoosmolality by excreting maximum volume of dilute urine, i.e., urine
osmolality will be < 100 mosmol/kg and specific gravity will be < 1.003. This occurs
in patients with primary polydipsia.

If primary polydipsia is not present, decreased plasma osmolality suggests impaired


free water excretion due to the action of AVP on the kidney. The secretion of AVP
may be a physiologic response to hemodynamic stimuli or it may be inappropriate
in the presence of hyponatremia and euvolemia.
SIADH is characterized by hyponatremia with decreased plasma osmolality and
concentrated urine (urine osmolality > 100 mosmol/kg and urine Na+ concentration
usually greater than 40 mmol/L). Urine Na+ excretion rate is equal to intake.
Patients are typically normovolemic.

Na+ is the major ECF cation and is largely restricted to this compartment.
Therefore, ECF volume contraction indicates a deficit in total body Na+ content.
Volume depletion in patients with normal underlying renal function results in
enhanced tubule Na+ reabsorption and a urine Na+ concentration < 20 mmol/L.

Urine Na+ concentration > 20 mmol/L in hypovolemic hyponatremia suggests


diuretic therapy, hypoaldosteronism, a salt-wasting nephropathy, or occasionally
vomiting.

30)

Which test is not useful for the diagnosis of hyponatraemia?

a.

Plasma electrolytes [ Plasma and urine electrolytes and osmolality are the only tests
required to diagnose the cause of hyponatraemia.

Initial laboratory evaluation also includes glucose, BUN, creatinine, and uric acid.]
b.Urine electrolytesc.Urine osmolality

d.

Plasma renin activity[ Measurement of plasma renin activity is useful when there is
doubt about clinical signs of ECF volume.]

e.

Plasma ADHT [ Plasma ADH is not very helpful in distinguishing between the causes
of hyponatraemic states. ADH is activated both in hypovolaemic and hypervolaemic
states. Most chronic hypervolaemic states (cardiac failure, cirrhosis and nephrotic
syndrome) have impaired circulation that activates ADH release through non-
osmotic mechanisms. Indeed, these disorders may have higher circulating ADH
levels than patients with SIADH. ADH is suppressed in primary polydipsia and
iatrogenic water intoxication.]31)Plasma ADH level is low ina.SIADHb.Cardiac
failurec.Cirrhosisd.Nephrotic syndrome

e.

Primary polydipsiaT

32)
What is the treatment of mild asymptomatic hyponatremia?a.Increase oral sodium
intake b.Stop loop diureticc.Loop diureticd.Water restriction

e.

No treatment T [ Mild asymptomatic hyponatremia requires no treatment. The rate


of correction of hyponatremia depends on the absence or presence of neurologic
dysfunction. In asymptomatic patients, the plasma Na+ concentration should not be
raised by more than 0.5 to 1.0 mmol/L per h and 10 to 12 mmol/L over the first 24
h.]

33)

What is the treatment of asymptomatic hyponatremia with ECF volume contraction?


a.Increase oral sodium intake b.Stop loop diuretic

c.

Give thiazided.Water restriction

e.

Isotonic salineT [ Give sodium as normal saline. The direct effect of the IV normal
saline on the plasma Na+ concentration is trivial. When euvolemia is restored by
normal saline, the stimulus for AVP release is removed allowing the excess free
water to be excreted. Acute or severe hyponatremia (plasma Na+ concentration
<110 to 115 mmol/L) usually present with altered mental status and/or seizures and
requires more rapid correction.]

34)

What is the treatment of asymptomatic hyponatremia with edema?

a.

Restrict Na+ intake[ The hyponatremia associated with edematous states reflect
the severity of the underlying disease. It is usually asymptomatic. These patients
have increased total body water that exceeds the increase in total body Na+
content. Treatment is restriction of Na+ and water intake.]

b.

Restrict water intake [ Dietary water intake should be less than the urine output.]

c.

Correct hypokalemia [ Correction of the K+ deficit may raise the plasma Na+
concentration by shifting of Na+ out of cells as K+ moves in.]
d.

Loop diuretic[ Use loop diuretics to increase water loss. Replace a proportion of the
urinary Na+ loss to ensure net free water excretion.]

e.

All of the aboveT

35)

Water restriction is useful in hyponatremia due toa.Cardiac failureb.Primary


polydipsiac.SIADHd.Renal failure

a.

All of the aboveT [ Water restriction will ameliorate all forms of hyponatremia, but it
is not the optimal therapy in all cases. Hyponatremias associated with the depletion
of extracellular-fluid volume require correction of the sodium deficit.]

36)

What is the treatment of symptomatic hyponatremia with dilute urine?

a.

Water restrictionT [ Patients with symptomatic hyponatremia and dilute urine


(osmolality < 200 mOsm per kilogram of water) but with less serious symptoms
usually require only water restriction and close observation.]

b.

Hypertonic saline [ Infusion of hypertonic saline is indicated only when severe


symptoms (e.g., seizures or coma) are present.]c.Oral sodiumd.Thiazide

e.

Furosemide

37)

50-year-old male admitted with seizures has a serum sodium level of 115 mEq/L.
What is the treatment?a.Oral sodium

b.Normal saline IV

c.

Hypertonic saline IV T [ Severe symptomatic hyponatremia should be treated with


hypertonic saline. Patients who have symptomatic hyponatremia with concentrated
urine (osmolality > 200 mOsm per kilogram of water) and clinical euvolemia or
hypervolemia require infusion of hypertonic saline. Hypertonic saline is usually
combined with furosemide to limit treatment induced expansion of the extracellular-
fluid volume. The plasma Na+ concentration should not be raised by more than 12
mEq/L during the first 24 h.]

d.

IV frusemide

FORMULAS FOR MANAGING HYPONATREMIA

† In addition to its complete distribution in the extracellular compartment, this


infusate induces osmotic removal of water from the intracellular
compartment.ESTIMATED TOTAL BODY WATER (IN LITERS)

Children = 0.6

Nonelderly men = 0.6

Nonelderly women = 0.5

Elderly men = 0.5

Elderly women = 0.45

Extracellular fluids = 40% of total body water

Intracellular fluids = 60% of total body water

38)

How much sodium is needed to correct Na+ concentration from 115 mEq/L to 125
mEq/L in a 60-kg man?

a.

90 mEq/L

b.

180 mEq/L

c.

360 mEq/LT [ The quantity of Na+ required to increase the plasma Na+
concentration by a given amount is estimated by multiplying the deficit in plasma
Na+ concentration by the total body water. Normally, total body water is 60% of
lean body weight in men (50% of lean body weight in women). In this question
sodium needed is 125 – 115 X 60 X 0.60 = 360 mEq/L.]
d.

720 mEq/L

e.

1440 mEq/L

39)

What is the treatment for severe hyponatraemia and convulsions?

a.

IV furosemideb.Dialysis

c.

Hypertonic sodium chlorideT [ The symptoms and treatment for hyponatraemia


depends on the rate of development and severity of hyponatraemia. If
hyponatraemia has developed rapidly (over hours to days), cerebral edema may
develop. In such circumstances, it is safe to correct the plasma sodium relatively
rapidly. Infusion of hypertonic (3%) sodium chloride solutions may be indicated,
especially when the patient is obtunded or convulsing.]d.Phenytoin 100mg every 8
hours

e.

Half-normal saline

40)

What is the treatment for severe asymptomatic chronic hyponatraemia?

a.

Hypertonic sodium chloride[ Rapid correction of hyponatraemia which has


developed slowly (over weeks to months) can cause brain damage (central pontine
myelinolysis). Cerebral cells adapt to slowly developing hyponatraemia and
hypoosmolality by reducing the intracellular osmolality, thus maintaining normal
cell volume. Rapid correction of hyponatraemia will cause an abrupt increase in
extracellular osmolality. Rapid increase in extracellular osmolality can lead to water
shifting out of the cerebral neurons, abruptly reducing their volume and risking
detachment from their myelin sheaths ('myelinolysis'). Therefore, rate of correction
of the plasma Na concentration in chronic asymptomatic hyponatraemia should not
exceed 10 mEq/L/day.]

b.
Water and salt restrictionT

c.

Demeclocycline[ Demeclocycline inhibits renal tubular responsiveness to ADH,


leading to increased water excretion. Onset of action is 3 to 6 days.]

d.

-receptor antagonists[ V

-receptor antagonists (conivaptan) will likely become the first-line treatment for
euvolemic and hypervolemic hypernatremia.]

e.

Furosemide [ Furosemide in combination with sodium chloride tablets will increase


urine volume and allow more water intake.]

41)

What is the cause of osmotic demyelination syndrome?a.Inadequate steroid in


multiple sclerosis

b.

Rapid correction of hyponatremiaT [ The risk of correcting hyponatremia too rapidly


is the development of the osmotic demyelination syndrome. See figure below.
Patients with chronic hyponatremia are most susceptible. Hypertonic saline can
cause sudden osmotic shrinkage of brain cells. It is characterized by flaccid
paralysis, dysarthria, and dysphagia. Diagnosis is confirmed by neuroimaging
studies. There is no specific treatment.]

c.

Fast diuresis in massive ascites

d.

More than 12 L of water intake in primary polydipsia[ Water restriction in primary


polydipsia may also lead to too rapid correction of hyponatremia. IV saline in ECF
volume-contracted patients may also lead to too rapid correction of hyponatremia.
It is due to AVP suppression and brisk water diuresis. It can be prevented by
administration of water or use of an AVP analogue to slow down the rate of free
water excretion.]e.Toxic effect of AVP

Within minutes after the development of hypotonicity, water gain causes swelling of
the brain and a decrease in osmolality of the brain. Partial restoration of brain
volume occurs within a few hours as a result of cellular loss of electrolytes (rapid
adaptation). The normalization of brain volume is completed within several days
through loss of organic osmolytes from brain cells (slow adaptation). Low osmolality
in the brain persists despite the normalization of brain volume. Proper correction of
hypotonicity reestablishes normal osmolality without risking damage to the brain.
Aggressive correction of hyponatremia can lead to irreversible brain damage.

42)

How does central pontine myelinolysis clinically present?

a.

QuadriplegiaT [ Central pontine myelinolysis typically presents as quadriplegia and


pseudobulbar palsy. Diagnosis is by MRI. Partial forms present as confusion,
dysarthria, and/or disturbances of conjugate gaze without quadriplegia. Most cases
are associated with rapid correction of hyponatremia or with

hyperosmolar states. The pathology is demyelination without inflammation in the


base of the pons.]b.Hyponatremiac.SIADH

d.

Intense thirst

43)

A previously healthy 30-year-old man has three generalized seizures two days after
an appendectomy. He was given diazepam and phenytoin intravenously and
undergoes laryngeal intubation with mechanical ventilation. Three liters of 5
percent dextrose in water had been infused during the first day after surgery. He
has subsequently drunk substantial amount of water. Clinically, he is euvolemic,
and he weighs 46 kg. He is stuporous and responds to pain but not to commands.
The serum sodium concentration is 112mEq per liter, the serum potassium
concentration is 4.1 mEq per liter, serum osmolality is 228 mOsm per kilogram of
water, and urine osmolality is 510 mOsm per kilogram of water. What is the
treatment?a.Water restriction

b.

Infusion of 3 percent sodium chloride

c.
Intravenous furosemide

d.

All of the aboveT [ This patient has hypotonic hyponatremia due to water retention
caused by the impaired excretion of water that is associated with the postoperative
state. The estimated volume of total body water is 23 liters (0.5 X 46). The retention
of 1 liter of 3 percent sodium chloride will increase the serum sodium concentration
by 16.7 mmol per liter (513 – 112) ÷ (23+1) =16.7.]

e.

None of the above

44)

A 58-year-old man with small-cell lung carcinoma presents with severe confusion
and lethargy. Clinically, he is euvolemic, and he weighs 60 kg. The serum sodium
concentration is 108 mEq per liter, the serum potassium concentration is 3.9 mEq
per liter, serum osmolality is 220 mOsm per kilogram of water, the serum urea
nitrogen concentration is 5 mg per deciliter, the serum creatinine concentration is
0.5 mg per deciliter per liter, and urine osmolality is 600 mOsm per kilogram of
water. What is the treatment?

a.

Water restriction

b.

Infusion of 3 percent sodium chloride

c.

IV furosemide

d.

All of the aboveT [ The diagnosis is tumor-induced syndrome of inappropriate


secretion of antidiuretic hormone on the basis of the presence of hypotonic
hyponatremia and concentrated urine in a euvolemic patient, the absence of a
history of diuretic use, and the absence of clinical evidence of hypothyroidism or
hypoadrenalism. The estimated volume of total body water is 36 liters. The
retention of 1 liter of 3 percent sodium chloride is estimated to increase the serum
sodium concentration by 10.9 mEq per liter {(513 – 108) ÷ (36+1) = 10.9}. The
initial goal is to increase the serum sodium concentration by 5 mEq per liter over
the next 12 hours. Therefore, 0.46 liter of 3 percent sodium chloride (5 ÷ 10.9), or
38 ml per hour, is required.]
e.

None of the above

45)

A 68-year-old woman is brought to the hospital because of progressive drowsiness


and syncope. She is being treated with a low-sodium diet and hydrochlorothiazide
daily for essential hypertension; she has had diarrhea for the past three days. She is
lethargic but has no focal neurologic deficits. She weighs 60 kg. Her blood pressure
while in a supine position is 96/56 mm Hg, and the pulse is 110 beats per minute.
The jugular veins are flat, and skin turgor is decreased. The serum sodium
concentration is 106 mEq per liter, the serum potassium concentration is 2.2 mEq
per liter, the serum urea nitrogen concentration is 46 mg per deciliter, the serum
creatinine concentration is 1.4 mg per deciliter, serum osmolality is 232 mOsm per
kilogram of water, and urine osmolality is 650 mOsm per kilogram of water. What is
the treatment?

a.

0.9% sodium chloride first followed by 0.45 % sodium chlorideT [Diagnosis is


hypotonic hyponatremia caused by thiazide therapy and gastrointestinal losses of
sodium. There is associated depletion of potassium also. Treatment is to withhold
hydrochlorothiazide and to infuse 0.9 percent sodium chloride solution containing
30 mEq of potassium chloride per liter. The estimated volume of total body water is
27 liters (0.45 X 60). The retention of 1 liter of this infusate will increase the serum
sodium concentration by 2.8 mEq per liter {(154 + 30) – 106 ÷ (27 + 1) = 2.8}. As
soon as the extracellular-fluid volume nears restoration, the nonosmotic stimulus to
arginine vasopressin release will cease resulting in rapid excretion of dilute urine
and correction of the hyponatremia at a rapid pace. Therefore, switch to 0.45
percent sodium chloride containing 30 mmol of potassium chloride per liter.]

b.

0.45 % sodium chloride first followed by 0.9 % sodium chloride

c.

5 % dextrose in water containing 30 mmol of potassium chloride

d.

3 percent sodium chloride

46)

What is the long-term treatment of asymptomatic hyponatremia that accompanies


cardiac failure?
a.

Water restriction T [Asymptomatic hyponatremia is common in edematous states


and in syndrome of inappropriate secretion of antidiuretic hormone. Hyponatremia
is due to a defect of water excretion. Water restriction (to <800 ml per day) is the
mainstay of long-term management. Goal is to induce negative water balance.]

b.

Thiazide diuretic[ Loop, but not thiazide, diuretics reduce urine concentration and
augment excretion of electrolyte-free water. Therefore, loop diuretics permit
relaxation of fluid restriction.]

c.

Loop diuretics with high sodium intake[ In the syndrome of inappropriate secretion
of antidiuretic hormone, but not in edematous disorders, loop diuretics should be
combined with plentiful sodium intake (in the form of dietary sodium or salt tablets).
This treatment increases water loss. If these measures fail, demeclocycline per day
can help by inducing nephrogenic diabetes insipidus.]

d.

Demeclocycline

47)

A 72-year-old woman presents with a 2-day history of presyncope when rising from
a chair. She has been taking hydrochlorothiazide, 25 mg/d, for 5 years for systolic
hypertension. Last week she had a bout of viral gastroenteritis with marked
diarrhea. She has been replacing the lost fluids by drinking 3 L of water per day.
When she rises from a seated position, her blood pressure drops 20 mm Hg. Serum
levels are as follows: sodium 128 mEq/L, potassium 3.1

mEq/L, creatinine 1.5 mg% and urea nitrogen 60mg%. Which is/are true statements
regarding this patient?

a.

ECF volume is contracted [ The patient has postural hypotension which indicates
ECF volume contraction. The most likely cause is gastrointestinal losses of salt and
water, with only water replacement. It is also likely that the thiazide diuretic is
contributing to the hyponatremia. Thiazides impair the kidney’s ability to reabsorb
sodium and to excrete free water.

b.
Release of AVP is stimulated[ ECF volume contraction from any cause (diarrhea,
vomiting, excessive sweating, diuretic use), stimulates the release of AVP. AVP
increases renal water reabsorption and ECF volume.]

c.

Low urine sodium concentration [ Volume contraction decreases renal perfusion


which stimulates renin release, and this causes the kidneys to avidly retain sodium.
The retention of water and sodium is appropriate in this setting and is supported by
a low urine sodium concentration (< 20 mmol/L) and a low urine volume. Often the
person who has a contracted ECF volume will drink water or another low-solute fluid
(e.g., tea), which contributes further to the hyponatremia. These patients have
serum sodium and body water levels that are lower than normal but have more loss
of sodium relative to loss of water.]

d.

She needs potassium replacement[ The ECF volume contraction, diarrheal losses
and diuretic use have resulted in hypokalemia in this patient. As ECF volume
contraction develops, the kidneys actively excrete potassium in exchange for
sodium in an attempt to preserve ECF volume. Volume restoration with normal
saline and potassium replacement is required until the postural drop in blood
pressure is less than 10 mm Hg. She should then be treated conservatively with oral
sodium and potassium replacement.]

e.

All of the aboveT [ The management of this patient should include temporary
discontinuation of the thiazide diuretic.]Water restriction will ameliorate all forms of
hyponatremia, but it is not the optimal therapy in allcases. Hyponatremias
associated with the depletion of extracellular-fluid volume require correction of the
sodium deficit. Isotonic saline is unsuitable for correcting the hyponatremia of the
syndrome of inappropriate secretion of antidiuretic hormone; if administered, the
resulting rise in serum sodium is both small and transient, with the infused salt
being excreted in concentrated urine and thereby causing a net retention of water
and worsening of the hyponatremia. Great vigilance is required in order to
recognize and diagnose hypothyroidism and adrenal insufficiency, since these
disorders tend to masquerade as cases of the syndrome of inappropriate secretion
of antidiuretic hormone. The presence of hyperkalemia should alwaysalert the
physician to the possibility of adrenal insufficiency.Whereas patients with persistent
asymptomatic hyponatremia require slow-paced management, those with
symptomatic hyponatremia must receive rapid but controlled correction.

You might also like